Vous êtes sur la page 1sur 77

INDEX

01-02-2019 ........................................................................................................................................................... 3

02-02-2019 .......................................................................................................................................................... 5

04-02-2019 ..........................................................................................................................................................8

05-02-2019 .........................................................................................................................................................11

06-02-2019 ........................................................................................................................................................ 14

07-02-2019......................................................................................................................................................... 17

08-02-2019 ....................................................................................................................................................... 20

09-02-2019 ........................................................................................................................................................24

11-02-2019.......................................................................................................................................................... 27

12-02-2019 .........................................................................................................................................................29

13-02-2019 ......................................................................................................................................................... 32

14-02-2019 ......................................................................................................................................................... 35

15-02-2019 .........................................................................................................................................................38

16-02-2019 ......................................................................................................................................................... 41

18-02-2019 .........................................................................................................................................................44

19-02-2019 ........................................................................................................................................................ 48

20-02-2019 ........................................................................................................................................................ 52

21-02-2019 ......................................................................................................................................................... 55

22-02-2019.........................................................................................................................................................58

23-02-2019......................................................................................................................................................... 61

25-02-2019 .........................................................................................................................................................64

26-02-2019......................................................................................................................................................... 67

27-02-2019 ......................................................................................................................................................... 70

28-02-2019 ........................................................................................................................................................ 73

2
www.shankariasacademy.com | www.iasparliament.com
01-02-2019 4) Which one of the following best describes
the term “Interchange Fee” sometimes seen in
1) “Project 75-I” sometimes seen in the news the news?
recently, is associated with which of the
following? a. The amount charged from the customers‘ for
using other banks‘ ATM services after a certain
a. Fighter aircrafts limit
b. Helicopters b. The amount charged by the agency to convert
c. Submarines foreign currency into a local currency

d. Armoured Fighting Vehicles c. The amount paid by the issuing bank to the
acquiring bank when the customer of the
issuer bank uses the latter‘s ATM
2) Consider the following statements with d. None of the above
respect to Strategic Partnership (SP) model in
Defence acquisition
1. It intends to encourage broader 5) Consider the following statements with
participation of the private sector in the respect to International Crops Research
manufacture of defence platforms in Institute for the Semi-Arid Tropics
India. 1. It is an international non-profit
2. Only four segments have been identified organization that undertakes scientific
for acquisition under this Strategic research for development.
Partnership (SP) route. 2. Its headquarters is located in Manila,
Which of the statement(s) given above is/are Philippines.
correct? Which of the statement(s) given above
a. 1 only is/are incorrect?

b. 2 only a. 1 only

c. Both 1 and 2 b. 2 only

d. Neither 1 nor 2 c. Both 1 and 2


d. Neither 1 nor 2

3) Consider the following statements with


respect to Annual Financial Statement 6) “Brumadinho dam” sometimes seen in the
1. It is not only necessary, but also news recently, is located in which of the
compulsory. following countries?

2. Though both the Houses of Parliament a. Columbia


discuss the Budget, only Lok Sabha b. Afghanistan
votes on it.
c. Australia
3. Vote-On-Account covers only the
expenditure to be incurred. d. Brazil
Which of the statement(s) given above is/are
correct? 7) “Naresh Chandra Task force” is associated
a. 2 only with which of the following?
b. 1 and 2 only a. Defence management
c. 1 and 3 only b. Issues with Start-ups
d. 1, 2 and 3 c. Regulate Fin-tech sector
d. None of the above

3
www.shankariasacademy.com | www.iasparliament.com
8) The term “Aber” sometimes seen in the news  It will provide a major boost to submarine
recently, refers to a design and manufacturing ecosystem in India
through the transfer of design, equipment
a. Local term for hangul deer
technology and skill sets.
b. Common currency for UAE and Saudi Arabia
2. c
c. Disputed Island between China and Taiwan
 Ministry of Defence‘s ambitious Strategic
d. Mountain in Atacama Desert Partnership (SP) model aims to provide a
significant fillip to the Government‘s „Make
in India‟ programme.
9) Consider the following statements with
respect to VAJRA Scheme  It envisages indigenous manufacturing of
major defence platforms by an Indian
1. It aims to reverse the brain drain Strategic Partner, who will collaborate with
process in India. foreign OEM to set up production facilities in
2. It comes under the Ministry of Human the country.
Resource Development.  The model has a long term vision of promoting
Which of the statement(s) given above is/are India as a manufacturing hub for defence
correct? equipment through transfer of niche
technologies and higher Indigenous Content
a. 1 only thereby enhancing self-sufficiency for meeting
b. 2 only the future requirements of the Armed Forces.

c. Both 1 and 2  It intends to institutionalise a transparent,


objective and functional mechanism
d. Neither 1 nor 2 to encourage broader participation of
the private sector, in addition to DPSUs /
OFB, in the manufacture of defence platforms
10) Central African Republic (CAR), sometimes and equipment such as aircraft, submarines,
seen in the news recently is bordered by which helicopters and armoured vehicles.
of the following?
 The following four segments have been
1. Kenya identified for acquisition under Strategic
2. Congo Partnership (SP) route:

3. Sudan 1. Fighter Aircraft

4. Atlantic Ocean 2. Helicopters

Select the correct answer using the code given 3. Submarines


below 4. Armoured Fighting Vehicles (AFVs) / Main
a. 2 and 3 only Battle Tanks (MBTs).

b. 1, 2 and 3 only 3. d

c. 1, 2, 3 and 4 only  Under Article 112 of the Constitution, a


Statement of Receipts and Payments
d. None (estimated) has to be tabled in the
Parliament for every financial year.

Answers  If the Budget is not passed within the


announced date, Article 116 of the Constitution
1. c empowers the Lok Sabha to pass the Vote-On-
Account, a document which covers only the
 The Defence Acquisitions Council (DAC) has expenditure incurred.
recently approved the indigenous construction
of six submarines worth 40,000 crore rupees  A ―vote-on-account‖ only deals with the
under project 75(India). expenditure in Government‟s budget,

4
www.shankariasacademy.com | www.iasparliament.com
while an ―interim budget‖ includes both  It aims to connect the Indian academic and
expenditure and receipts. research and development (R&D) systems to
the best of global science and scientists for a
4. c
sustained international collaborative research.
 Interchange fee is the amount paid by the
 It is a dedicated program exclusively for
issuing bank to the acquiring bank when the
overseas scientists and academicians with
customer of the issuer bank uses the latter‘s
emphasis on Non-resident Indians (NRI) and
ATM.
Persons of Indian Origin (PIO) / Overseas
 Increasing maintenance cost is forcing Indian Citizen of India (OCI) to work as adjunct /
Public Sector Banks to shut down ATMs, but visiting faculty for a specific period of
doing so will only add to their woes as they will time in Indian Public funded academic
consequently incur higher interchange fee. and research institutions.
 This is because they are the biggest issuers of 10. a
debit cards and cardholders will now be forced
 The United Nations Security Council has
to use other banks‘ ATMs.
recently adopted a resolution to extend
5. b sanctions against the Central African
Republic (CAR) for another year.
 It is headquartered in Patancheru
(Hyderabad, Telangana, India).  CAR is a landlocked country in Central
Africa.
6. d
 It is bordered by Chad to the north, Sudan to
 The Brumadinho dam disaster has occurred the northeast, South Sudan to the east, the
recently, when a tailings dam at an iron ore Democratic Republic of the Congo to the
mine in Brumadinho, Minas south, the Republic of the Congo to the
Gerais, Brazil suffered a catastrophic failure. southwest and Cameroon to the west.
 Global turmoil in the iron ore market followed
by this disaster may have come as a blessing
for India‟s ore mining and pellet- 02-02-2019
making companies.
1) Consider the following statements with
7. a respect to Pradhan Mantri Kisan Samman
Nidhi (PM-KISAN)
 In June 2011, government of India had
announced setting up a high-powered task 1. Under this scheme, farmers having
force headed by Naresh Chandra, to review cultivable land up to 5 hectares will be
the defense management in the country provided income support at the onset of
and make suggestions for implementation of every harvest year.
major defense projects.
2. Funded by the Government of India,
8. b this income support will be transferred
directly into the bank accounts of
 The central banks of the UAE and Saudi beneficiaries.
Arabia have launched a common digital
currency called "Aber", which will be used Which of the statement(s) given above is/are
in financial settlements between the two correct?
countries through Blockchains and
a. 1 only
Distributed Ledgers technologies.
b. 2 only
9. d
c. Both 1 and 2
 The Science and Engineering Research Board
(SERB), a statutory body of the Department of d. Neither 1 nor 2
Science and Technology (DST) launched the
‗Visiting Advanced Joint Research (VAJRA)
Faculty Scheme‘.

5
www.shankariasacademy.com | www.iasparliament.com
2) INSTEX Channel sometimes seen in the 2. It will provide beneficiaries, an assured
news recently, is a monthly pension from the age of 60
years.
a. Payment channel with Iran to circumvent U.S
sanctions Which of the statement(s) given above
is/are incorrect?
b. Passage that connects South China Sea with
East China Sea a. 1 only
c. Route to divert investments in Stock exchanges b. 2 only
based on algorithms
c. Both 1 and 2
d. World‘s busiest shipping traffic channel
d. Neither 1 nor 2
located in Red sea

6) Consider the following statements with


3) “CAR – T Therapy”, sometimes seen in the
respect to Fiscal Deficit (FD)
news recently, treats which of the following
diseases? 1. It is the difference between the Revenue
Receipts plus Non-debt Capital Receipts
a. Tuberculosis
(NDCR) and the total expenditure.
b. Cancer
2. It is reflective of the total borrowing
c. AIDS requirements of Government.
d. None of the above Which of the statement(s) given above is/are
correct?
a. 1 only
4) Consider the following statements with
respect to Railguns sometimes seen in the news b. 2 only
1. It uses electricity instead of gunpowder c. Both 1 and 2
to launch projectiles.
d. Neither 1 nor 2
2. It eliminates the need to store
explosives aboard warships.
7) The GoI has recently announced that a
3. United States has recently developed the
committee will be constituted to complete the
world‟s first ship-mounted
task of identifying De-Notified, Nomadic and
electromagnetic Rail gun.
Semi-Nomadic Communities which are not yet
Which of the statement(s) given above is/are formally classified, under which of the
correct? following organisations?
a. 2 only a. Ministry of Tribal affairs
b. 1 and 2 only b. Department of Social Justice
c. 2 and 3 only c. National Human Rights Commission
d. All of the above d. NITI Aayog

5) Consider the following statements with 8) The Renke Commission and


respect to Pradhan Mantri Shram-Yogi the Idate Commission sometimes seen in the
Maandhan news recently is associated with which of the
following?
1. It is a mega pension scheme for the
unorganised sector workers with yearly a. De-notified and Nomadic tribes
income up to Rs. 50,000.
b. 4R approach in Banking
c. Artificial Intelligence

6
www.shankariasacademy.com | www.iasparliament.com
d. Disaster Response  This programme will be funded by
Government of India.

9) Consider the following statements with  Around 12 crore small and marginal farmer
respect to Rashtriya Kamdhenu Aayog families are expected to benefit from this.

1. It aims to upscale sustainable genetic 2. a


up-gradation of cow resources and to  Germany, France and the UK has set up a
enhance production and productivity of payment channel with Iran called INSTEX, to
cows. help continue trade and circumvent US
2. It will look after effective sanctions.
implementation of laws and welfare 3. b
schemes for cows.
 An 11-year-old has become the first patient to
Which of the statement(s) given above is/are receive CAR-T therapy (immunotherapy) that
correct? uses the body's own cells to fight cancer.
a. 1 only  CAR-T is a personalized form of cancer
b. 2 only treatment.
c. Both 1 and 2  CAR-T involves removing immune cells and
modifying them in a laboratory so they can
d. Neither 1 nor 2 recognize cancer cells.
 Immunotherapy is treatment that uses your
10) Consider the following statements with body's own immune system to help fight
respect to Rashtriya Gokul Mission cancer.
1. It aims to conserve and develop  First, the patient has blood removed and the
indigenous breeds of cattle. white blood cells are separated out, with the
rest of the blood being returned to the patient.
2. It is run by the Ministry of Agriculture &
Farmers Welfare.  A harmless virus is used to insert genes into T-
Which of the statement(s) given above is/are cells, a special type of immune cell.
correct?  These genes cause the T-cells to add a hook on
a. 1 only to their surface, known as a chimeric antigen
receptor (CAR).
b. 2 only
 These engineered CAR-T cells - programmed
c. Both 1 and 2 to recognize and destroy the patient's cancer
d. Neither 1 nor 2 cells - are multiplied in huge numbers and
then infused back into the patient.
4. b
Answers
 The world‘s first ship-mounted
1. b electromagnetic railgun which is developed by
the Chinese Navy and can strike targets
 Under this programme, vulnerable more than 200 km away at seven times the
landholding farmer families, having cultivable speed of sound was tested in January 2018 and
land up to 2 hectares, will be provided direct will be ready for deployment by 2025.
income support at the rate of Rs. 6,000 per
year.  It use electricity instead of gunpowder
to launch projectiles, and therefore,
 This income support will be eliminates the need to store explosives
transferred directly into the bank aboard warships.
accounts of beneficiary farmers, in three
equal instalments of Rs. 2,000 each.  Railguns have long been touted as one of the
future technologies of warfare, with the ability

7
www.shankariasacademy.com | www.iasparliament.com
to launch projectiles at Mach 6 - or, more than  The Board shall ensure that special strategies
4,500 miles per hour. are designed and implemented to serve these
hard to-reach communities.
 The Chinese railgun is believed to be capable
of launching projectiles at nearly 5,600 miles 8. a
per hour.
 The GoI will setup a Committee under NITI
 Because the missile is fired using kinetic Aayog to complete the task of identifying De-
energy, this eliminates the risks notified, Nomadic and Semi-Nomadic
associated with keeping explosives on a communities not yet formally classified.
ship.
 The Nomadic and Semi-Nomadic communities
 U.S had a similar project until last year but move from place to place in search of a
scrapped its programme citing concerns about livelihood.
the colossal amount of power required to fire
them.  These communities are hard to reach, less
visible, and therefore, frequently left-out.
5. a
 The Renke Commission and the Idate
 Government of India in its recent interim Commission has done commendable work to
budget proposes to launch a mega pension identify and list these communities.
yojana namely 'Pradhan Mantri Shram-
Yogi Maandhan' for the unorganised 9. c
sector workers with monthly income upto  Rashtriya Kamdhenu Aayog aims to
Rs. 15,000. upscale sustainable genetic upgradation of cow
 This pension yojana shall provide them an resources and to enhance production and
assured monthly pension of Rs. 3,000 from the productivity of cows.
age of 60 years on a monthly contribution of a  The Aayog will also look after effective
small affordable amount during their working implementation of laws and welfare schemes
age. for cows.
 The Government will deposit equal matching 10. c
share in the pension account of the worker
every month.  The Budget for the year 2019-20 has increased
the allocation for Rashtriya Gokul Mission to
6. c Rs. 750 crore.
 Fiscal deficit for 2019-20 has been pegged at  It is a scheme, run by the Ministry of
3.4% of GDP. Agriculture & Farmers Welfare aims to
 Target of 3% of fiscal deficit to be achieved by conserve and develop indigenous breeds of
2020-21. cattle.

 Fiscal deficit brought down to 3.4% in 2018-19


RE from almost 6% seven years ago. 04-02-2019
7. d 1) Consider the following statements with
 The GoI will setup a Committee under NITI respect to Know My India Programme
Aayog to complete the task of identifying De- 1. It is an initiative for Diaspora
notified, Nomadic and Semi-Nomadic engagement which familiarizes Indian-
communities not yet formally classified. origin youth with their Indian roots and
 The Government will also set-up a Welfare contemporary India.
Development Board under the Ministrsy of 2. It is initiated by the Ministry of External
Social Justice and Empowerment specifically affairs.
for the purpose of implementing welfare and
development programmes for De-notified, Which of the statement(s) given above is/are
Nomadic and Semi-Nomadic communities. correct?
a. 1 only

8
www.shankariasacademy.com | www.iasparliament.com
b. 2 only 5) Consider the following statements with
respect to Government e-Marketplace (GeM)
c. Both 1 and 2
1. It aims to enhance transparency,
d. Neither 1 nor 2
efficiency and speed in public
procurement.
2) Consider the following statements with 2. In its Interim Budget 2019-20, GoI has
respect to Bharat Rang Mahotsav (BRM) decided to restrict this platform to only
public sector enterprises which have
1. It is an annual International theatre annual net profit of over Rs. 50,00
festival of India. crore.
2. It is being organized by the National Which of the statement(s) given above is/are
School of Drama (NSD). correct?
Which of the statement(s) given above is/are a. 1 only
correct?
b. 2 only
a. 1 only
c. Both 1 and 2
b. 2 only
d. Neither 1 nor 2
c. Both 1 and 2
d. Neither 1 nor 2
6) “Lao-Ong Mo” is a post-harvest festival
celebrated by which of the following tribes?
3) Which of the following states has recently a. Mundas
became the first State to set up a price
monitoring and research unit (PMRU) to track b. Bhil
violation of prices of essential drugs and
c. Konyak Naga
medical devices under the Drugs Price Control
Order (DPCO)? d. Lepchas
a. Kerala
b. Tamilnadu 7) Consider the following statements with
respect to Group of Monuments at Hampi
c. Odisha
1. It is located in the Tungabhadra basin,
d. West Bengal
Karnataka.
2. Hampi was the last capital of the last
4) Consider the following statements great Hindu Kingdom of Vijayanagar.
1. India has 28 species of turtles, of which 3. It is a UNESCO World Heritage Site.
20 are found in Assam.
Which of the statement(s) given above
2. The black softshell turtle widely found is/are incorrect?
along Indian coast is listed as critically
a. 2 only
endangered under the IUCN Red list.
b. 2 and 3 only
Which of the statement(s) given above is/are
correct? c. 1, 2 and 3
a. 1 only d. None
b. 2 only
c. Both 1 and 2
d. Neither 1 nor 2

9
www.shankariasacademy.com | www.iasparliament.com
8) Consider the following statements with  The main objective of the Foundation is to
respect to Battle of Chausa provide assistance to the children / youth
rendered orphan / destitute in communal,
1. It was fought between Sher shah suri
caste, ethnic or terrorist violence for
and the Mughal Emperor Humayun in
their rehabilitation besides promoting
1539.
communal harmony and national integration
2. At the end of this battle, Humayun left through various activities.
India and went into exile.
 Know India Programme (KIP) is a
Which of the statement(s) given above is/are flagship initiative for Diaspora engagement
correct? which familiarizes Indian-origin youth (18-30
a. 1 only years) with their Indian roots and
contemporary India, through a three-week
b. 2 only orientation programme organised by
the Ministry of External affairs.
c. Both 1 and 2
2. c
d. Neither 1 nor 2
 Bharat Rang Mahotsav (BRM),
the annual international theatre festival of
9) Rivers Devika and Tawi flows in which of India is organized by the National School of
the following states? Drama (NSD), was established two decades
ago to stimulate the growth and development
a. Karnataka
of theatre across India.
b. Jammu and Kashmir
 Originally a national festival showcasing the
c. Meghalaya work of the most creative theatre workers in
India, BRM has evolved to international scope,
d. Gujarat
hosting theatre groups from around the world,
and is now the largest theatre festival of
Asia.
10) Pobitora wildlife Sanctuary is located in
which of the following states?  20th Bharat Rang Mahotsav was recently
a. Assam inaugurated in New Delhi.

b. Arunachal Pradesh 3. a

c. Meghalaya  Kerala has become the first State to set up a


price monitoring and research unit (PMRU) to
d. Mizoram track violation of prices of essential drugs and
medical devices under the Drugs Price Control
Order (DPCO).
Answers 4. a
1. d  The black softshell turtle (Nilssonia
 Know My India Programme is a unique nigricans) figures in the International Union
programme initiated by the National for Conservation of Nature‘s (IUCN) Red List
Foundation for Communal Harmony as ―extinct in the wild‖.
(NFCH) to bring together financially assisted  But a few temple ponds in Assam and
children of the Foundation from different Bangladesh are bringing these turtles back
States/Regions of the country to promote from the brink.
oneness, fraternity and national integration.
 Being received the divine protection for many
 The NFCH is an autonomous organization years, the rarest of India‘s 28 turtle species is
under the administrative control of back where it belongs (in the wild).
the Ministry of Home Affairs.

10
www.shankariasacademy.com | www.iasparliament.com
 Recently, 35 hand-reared turtle hatchlings, 05-02-2019
including 16 black softshells, in the Haduk
Beel (wetland) of Pobitora Wildlife Sanctuary. 1) Consider the following statements with
respect to VISION 2030, sometimes seen in the
5. a news recently.
 Through budget 2019-20, Government e- 1. It expresses government‟s vision for the
Marketplace (GeM) is now being extended next two decades, listing thereby ten
to all CPSEs. most important dimensions in 2030.
6. c 2. It was launched by the NITI Aayog.
 ―Lao-Ong Mo‖ is a post-harvest festival of Which of the statement(s) given above is/are
the Konyak Naga tribe inhabiting the correct?
district of Mon, Nagaland, running along the
a. 1 only
Myanmar border.
b. 2 only
7. d
c. Both 1 and 2
 The austere, grandiose site of Hampi was
the last capital of the last great Hindu d. Neither 1 nor 2
Kingdom of Vijayanagar.
 Group of Monuments at Hampi, a UNESCO 2) Consider the following statements with
World Heritage Site is located in respect to Prompt Corrective Action (PCA)
the Tungabhadra basin in Bellary District, framework
Central Karnataka.
1. It is imposed on banks whose capital,
8. a asset quality, profitability and/or
 Battle of Chausa was fought between leverage do not meet pre-specified
Humayun and Sher Shah Suri in 1539. thresholds.

 The Mughal forces under Humayun were 2. It is applicable only to commercial


defeated by Sher Shah. Humayun narrowly banks and not extended to co-operative
escaped. banks, non-banking financial
companies (NBFCs) and FMIs.
 Battle of Kannauj in which, Mughal forces
once again clashed with Sher Shah in May Which of the statement(s) given above is/are
1540. correct?
a. 1 only
 This time Humayun was decisively defeated by
Sher Shah. b. 2 only
 Humayun left India and went into exile. He c. Both 1 and 2
remained in exile for the next 15 years.
d. Neither 1 nor 2
9. b
 Recently, Prime Minister has laid foundation
3) “Periodic Labour Force Survey”, sometimes
stone of the project aimed at reducing
seen in the news recently, was prepared by
pollution of Rivers Devika & Tawi flows in
which of the following?
Jammu and Kashmir, under the National
River Conservation Plan (NRCP). a. Labour Bureau
 This project is scheduled for completion by b. Society for Labour and Development
March 2021. c. NITI Aayog
10. a d. National Sample Survey Office
 Recently, 35 hand-reared turtle hatchlings,
including 16 black softshells, in the Haduk
Beel (wetland) of Pobitora Wildlife Sanctuary.

11
www.shankariasacademy.com | www.iasparliament.com
4) Senkaku Islands or the Daioyu 8) Asian Infrastructure Finance 2019 report is
Islands sometimes seen in the news recently, is released by which of the following?
located in which of the following?
a. Asian Development Bank
a. South China Sea
b. New Development Bank
b. Pacific Ocean
c. Asian Infrastructure Investment Bank
c. Sea of Japan
d. None of the above
d. East China Sea

9) Shekatkar Committee sometimes seen in the


5) Consider the following statements with news recently, is associated with?
respect to World Wetlands Day
a. Rebalancing defence expenditure
1. It was celebrated every year to
b. Universal Health Coverage
commemorate the adoption of the
Convention on Wetlands in the Iranian c. Reservation of women in legislative bodies
city of Ramsar, located on the shores of
the Caspian Sea. d. None of the above

2. The theme for the year 2019 is


„Wetlands and Climate Change‟. 10) Consider the following pairs
Which of the statement(s) given above is/are 1. Varahi hydel project – Telangana
correct?
2. Kiru Hydroelectric project –
a. 1 only Maharashtra
b. 2 only Which of the pair(s) given above is/are
c. Both 1 and 2 correctly matched?

d. Neither 1 nor 2 a. 1 only


b. 2 only

6) “Pink Sarathi Initiative” sometimes seen in c. Both 1 and 2


the news recently, is going to be launched in d. Neither 1 nor 2
which of the following cities?
a. Chennai
Answers
b. Bengaluru
1. d
c. Mumbai
 The Government has recently unveiled its
d. Kolkata vision for the next decade, listing thereby ten
most important dimensions in 2030.
7) “Miyako strait” sometimes seen in the news  This comprehensive ten-dimensional Vision
recently, separates which of the following two will create an India where poverty,
Islands? malnutrition, littering and illiteracy would be a
matter of the past.
a. Miyako and Osumi
 The vision was presented by the Indian
b. Borneo and Sulawesi
Finance Minister during his Budget
c. Miyako and Okinawo speech recently in Parliament.
d. Chukchi Peninsula and Cape Prince of Wales 2. c
 The RBI has specified certain regulatory
trigger points, as a part of PCA Framework for

12
www.shankariasacademy.com | www.iasparliament.com
initiation of certain structured and  On the contrary, China also claimed these
discretionary actions in respect of banks islands and named them as Diaoyu Islands.
hitting such trigger points.
 Currently, these Islands are administered by
 Under the revised PCA framework: Capital, Japan.
Asset quality and Profitability continue to be
the key areas for monitoring.  Recently, large ships of the Japanese Coast
Guard keep a tight round- the-clock vigil on
 Indicators to be tracked for Capital, asset this group of Islands.
quality and profitability would be
CRAR/ Common Equity Tier I ratio, Net  Their mission is to ensure that Japan
NPA ratio (NNPA) and Return on Assets (RoA) maintains its hold over these small but
respectively. strategically significant islands in the East
China Sea.
 Leverage would be monitored
additionally as part of the PCA framework. 5. c

 The PCA framework is applicable only to  World Wetlands Day is celebrated every year
commercial banks and not extended to co- on 2 February.
operative banks, non-banking financial  This day marks the date of the adoption of the
companies (NBFCs) and FMIs. Convention on Wetlands on 2 February 1971,
3. d in the Iranian city of Ramsar on the shores of
the Caspian Sea.
 The National Sample Survey
Office‟s Periodic Labour Force Survey showed  The theme for 2019 is ‗Wetlands and
that India‘s labour force participation rate, Climate Change‟.
which is the proportion of the population 6. b
working or seeking jobs, fell to 49.8% in 2017-
‘18 from 55.9% in 2011-‘12.  The Bangalore Metropolitan Transport
Corporation (BMTC) will soon have a fleet of
4. d 25 Pink Sarathi vehicles on the city‘s
roads to attend to the complaints of
women passengers travelling in its buses.
 The vehicles were procured under the Central
government‘s Nirbhaya scheme.
7. c

 At the heart of the dispute between the China


and Japan are eight uninhabited islands and
rocks in the East China Sea.
 The Japanese call them Senkaku, and
designate them as sovereign territory.  The 250 km wide passageway Miyako
Strait between the Miyako and Okinawa

13
www.shankariasacademy.com | www.iasparliament.com
islands is seen as a major choke-point inclusive and equitable quality
through which China‘s People‘s Liberation education at all levels.
Army Navy can channel into the West Pacific,
2. It subsumes three erstwhile Schemes of
dominated by the U.S. Navy.
Sarva Shiksha Abhiyan (SSA), Rashtriya
 Recently, large ships of the Japanese Coast Madhyamik Shiksha Abhiyan (RMSA)
Guard keep a tight round- the-clock vigil on and Teacher Education (TE).
this group of Islands.
Which of the statement(s) given above is/are
 Their mission is to ensure that Japan correct?
maintains its hold over these small but a. 1 only
strategically significant islands in the East
China Sea. b. 2 only
8. c c. Both 1 and 2
9. a d. Neither 1 nor 2
 The Committee of Experts (CoE) constituted
by the Ministry of Defence under the 2) Consider the following statements with
chairmanship of Lt. Gen (Retd) respect to Rashtriya Vayoshri Yojana (RVY)
DB Shekatkar to recommend measures
to enhance combat 1. It will provide physical aids and
capability and rebalance defence assisted-living devices for senior
expenditure of the armed forces, submitted citizens belonging to BPL category.
its report in December 2016. 2. It is being implemented through the
10. d Artificial Limbs Manufacturing
Corporation (ALIMCO).
 Recently, PM has laid the foundation stone of
624 MW Kiru Hydroelectric project in Which of the statement(s) given above is/are
Kishtwar, Jammu Kashmir. correct?

 It is a run of river project across River Chenab a. 1 only


and once completed, the project will generate b. 2 only
2272 Million Units of electricity annually.
c. Both 1 and 2
 PM has also inaugurated the 9 MW Dah
Hydroelectric project. d. Neither 1 nor 2

 Located in Dah near village Datang, this


project is a run-of–the-river scheme, 3) Consider the following statements with
by Jammu & Kashmir State Power respect to Pre-Departure Orientation Training
Development Corporation Limited. (PDOT) Programme
 Tremors of light intensity were experienced in 1. It provides requisite Pre-Departure
villages in the vicinity of Varahi hydro- Orientation and aims to enhance the
electric project in Hosanagar and soft skills of potential emigrants.
Thirthahalli taluks of Shivamogga
district, Karnataka. 2. The Ministry of External Affairs in
collaboration with Ministry of Skill
Development and Entrepreneurship is
conducting the programme.
06-02-2019
3. National Skill Development Corporation
1) Consider the following statements with (NSDC) is the implementing agency for
respect to Samagra Shiksha Abhiyan this programme.
1. It envisages school education as a Which of the statement(s) given above
continuum from pre-school to senior is/are incorrect?
secondary level and aims to ensure
a. 2 only

14
www.shankariasacademy.com | www.iasparliament.com
b. 3 only Which of the statement(s) given above is/are
correct?
c. 2 and 3 only
a. 1 only
d. None
b. 2 only
c. Both 1 and 2
4) Consider the following statements with
respect to employment of women in Indian d. Neither 1 nor 2
mines
1. The Mines Act, 1952 prohibited the
7) Consider the following statements with
employment of women in any part of a
respect to Earthquake swarm
mine.
1. It is a series of low magnitude
2. Recently, the Ministry of Mines has
earthquakes without a discernible main
issued a notification, easing the
shock.
restrictions and allowed women to work
in mines. 2. It occurs over a period of time ranging
from days, weeks to even months.
Which of the statement(s) given above is/are
correct? Which of the statement(s) given above is/are
correct?
a. 1 only
a. 1 only
b. 2 only
b. 2 only
c. Both 1 and 2
c. Both 1 and 2
d. Neither 1 nor 2
d. Neither 1 nor 2

5) Consider the following statements with


respect to GSAT-31 8) Global Crisis of Nuclear Waste
Report sometimes seen in the news recently,
1. It is an imaging satellite meant
was published by which of the following
exclusively for military purposes.
agencies?
2. It was launched using Polar Satellite
a. Atomic Energy Commission
Launch Vehicle (PSLV) C-45.
b. International Energy Agency
Which of the statement(s) given above is/are
correct? c. International Atomic Energy Agency
a. 1 only d. Green Peace
b. 2 only
c. Both 1 and 2 9) Consider the following statements with
respect to President‟s address to the
d. Neither 1 nor 2
Parliament
1. Originally, the Constitution required the
6) Consider the following statements with President to address both Houses of
respect to Pravasi Kaushal Vikas Yojana Parliament at the commencement of
every session.
1. It aims to enable a large number of
youths to take up industry-relevant skill 2. The speech will be drafted by the
training that will help them in securing President‟s Secretariat Office.
a better livelihood in India.
Which of the statement(s) given above is/are
2. The scheme has been implemented by correct?
National Skill Development Corporation
a. 1 only
(NSDC).

15
www.shankariasacademy.com | www.iasparliament.com
b. 2 only Corporation (ALIMCO)‖, a Public Sector
Undertaking under the Ministry of SJ&E, as
c. Both 1 and 2
the sole Implementing Agency.
d. Neither 1 nor 2
3. d
 PDOT Programme provides requisite Pre-
10) Consider the following pairs Departure Orientation (PDO) with regard to
language, culture, do‘s and don‘ts in the
1. Arabithittu WLS – Karnataka
destination country and aims to enhance the
2. Tungareshwar WLS – Telangana soft skills of potential emigrants.
3. Thane Creek Flamingo WLS –  The Ministry of External Affairs (MEA) in
Maharashtra collaboration with Ministry of Skill
Development and
Which of the pair(s) given above is/are
Entrepreneurship (MSDE) is conducting
correctly matched?
the PDOT programme under the Pravasi
a. 2 only Kaushal Vikas Yojna (PKVY).
b. 1 and 2 only  The National Skill Development
Corporation (NSDC) is the implementing
c. 1 and 3 only
agency for this programme.
d. 1, 2 and 3
4. d
 Recently, the Ministry of Labour has lifted
Answers the time bar on the employment of women
in mines across the country.
1. c
 Previously Section 46 of the Mines Act, 1952
 Department of School Education and Literacy prohibited the employment of any women in
has launched an Integrated Scheme for School any part of a mine which is below-ground and
Education called Samagra Shiksha on ground, except between 6 a.m. and 7
Abhiyan. p.m.
 It subsumes three erstwhile Centrally 5. d
Sponsored Schemes of Sarva Shiksha Abhiyan
(SSA), Rashtriya Madhyamik Shiksha Abhiyan  India‘s telecommunication satellite,
(RMSA) and Teacher Education (TE). GSAT-31 was successfully launched on
February 06, 2019 from Kourou launch
 The new integrated scheme envisages school base, French Guiana by Ariane-5 VA-247.
education as a continuum from pre-school to
senior secondary level.  The satellite provides Indian mainland and
island coverage.
2. c
 With a mission life of around 15 years, GSAT-
 The Ministry of Social Justice and 31 will be used for supporting VSAT networks,
Empowerment has introduced a scheme for Television uplinks, Digital Satellite News
providing physical aids and Assisted Living Gathering, DTH-television services, cellular
Devices for Senior Citizens belonging to BPL backhaul connectivity and many such
category named ―Rashtriya Vayoshri Yojana applications.
(RVY)‖ on 1st April, 2017.
6. b
 It provides Senior Citizens, belonging to BPL
category and suffering from age related  Pravasi Kaushal Vikas Yojana (PKVY) is aimed
disabilities/ infirmities, with such physical aids at providing suitable skill sets for those
and assisted living devices which can restore seeking employment abroad by training
near normalcy in their bodily functions. them in different trades and also imparting
soft skills.
 The Scheme is being implemented through the
―Artificial Limbs Manufacturing

16
www.shankariasacademy.com | www.iasparliament.com
 PKVY is a skill development initiative by the  Article 87(1) originally required the President
Ministry of External Affairs. to address both Houses of Parliament at the
commencement of every session.
 The MEA and the Ministry of Skill
Development and Entrepreneurship (MSDE)  The Constitution (First Amendment) Act,
had signed a Memorandum of Understanding 1951, amended this provision.
(MoU) for its implementation.
 Article 87(1) says: ―At the commencement of
 The scheme will be implemented by National the first session after each general election to
Skill Development Corporation the House of the People and at the
(NSDC) through its training partners in commencement of the first session of each year
consultation with Ministry of External Affairs the President shall address both Houses of
and Ministry of Skill Development and Parliament assembled together and inform
Entrepreneurship. Parliament of the causes of its summons.
7. c  The President‘s Address is the statement of
policy of the Government and as such
 Earthquake swarm is a series of many
is drafted by the Government.
(sometimes thousands) low magnitude
earthquakes without a discernible  It is not the President but the Government
main shock. which is responsible for the contents of the
Address.
 They occur in a localised region and over a
period of time ranging from days, weeks to 10. c
even months, without a clear sequence of
foreshocks, main quakes and aftershocks.  The Mumbai-Ahmadabad high speed (Bullet)
train corridor project was accorded wildlife
 When seismic energy piles up inside the Earth clearance recently.
and is released in small amounts from certain
points, such a series of earthquakes can occur.  The project will consume lands from
the Thane Creek Flamingo (TCF)
 Sometimes, these are also accompanied by Wildlife Sanctuary, Maharashtra.
acoustic or sound emissions.
 Apart from the creek, the project also involves
 Dahanu town in Maharashtra‟s Palghar diverting forestland and non-forestland from
district has been hit by some 30 low-intensity the Sanjay Gandhi National Park which houses
earthquakes since November 2018. leopards and the Tungareshwar Wildlife
Sanctuary, Maharashtra.
 Palghar was rocked by a series of minor
earthquakes (6 tremors) on a single day  Sri Chamarajendra Zoological Gardens in
recently. Mysuru has successfully translocated 52
spotted deer that were in surplus in its
 The magnitudes of the quakes ranged between collection to Arabithittu wildlife
3 and 4.1 on the Richter scale. sanctuary in Karnataka.
 Many aftershocks of lesser magnitude have
also been observed in the area.
07-02-2019
 Data collected so far point to an
“earthquake swarm”. 1) Consider the following statements with
respect to Agri-Market Infrastructure Fund
8. d (AMIF)
 The Global Crisis of Nuclear Waste is a 1. It is a corpus of Rs. 2,000 crore under
report published by Greenpeace France in the National Bank for Agriculture and
January 2019. Rural Development (NABARD).
 An analysis of waste storage facilities in seven 2. It was created for the development and
countries with nuclear power revealed up-gradation of agricultural marketing
that several were near saturation. infrastructure in rural and regulated
9. a wholesale markets.

17
www.shankariasacademy.com | www.iasparliament.com
Which of the statement(s) given above is/are a. UNESCO and World Bank
correct?
b. UNICEF and ILO
a. 1 only
c. World Bank and WEF
b. 2 only
d. UN and Amnesty International
c. Both 1 and 2
d. Neither 1 nor 2
5) Consider the following statements with
respect to Rebate of State Levies (RoSL)
Scheme
2) Consider the following statements with
respect to Darwaza Band -Part 2' campaign 1. It is a subsidy scheme which involves
refund of taxes and levies paid by
1. It was launched by Swachh Bharat
exporters.
Mission Grameen across the country.
2. Currently, it is applicable only to
2. It focuses on sustaining the open
exporters of apparels and made-ups.
defecation free status of villages across
the country. Which of the statement(s) given above is/are
correct?
3. It has been supported by the World
Bank. a. 1 only
Which of the statement(s) given above is/are b. 2 only
correct
c. Both 1 and 2
a. 3 only
d. Neither 1 nor 2
b. 1 and 2 only
c. 1 and 3 only
6) Consider the following statements with
d. 1, 2 and 3 respect to Indo-Saracenic architecture
1. It is a revival architectural style used by
British architects in India in the 19th
3) Consider the following statements with
century.
respect to North Atlantic Treaty Organization
(NATO) 2. The Madras High Court buildings are an
example of this style.
1. It is an alliance of 29 countries only
from North America and Europe. Which of the statement(s) given above is/are
correct?
2. The fundamental role of NATO is to
safeguard the freedom and security of a. 1 only
its member countries by political and
b. 2 only
military means.
c. Both 1 and 2
Which of the statement(s) given above is/are
correct? d. Neither 1 nor 2
a. 1 only
b. 2 only 7) “Exercise RAHAT” sometimes seen in the
news recently, is associated with
c. Both 1 and 2
a. Oil spillage
d. Neither 1 nor 2
b. Rehabilitation of Naxal
c. Humanitarian assistance
4) The Report on Social Protection for
Children sometimes seen in the news recently, d. Protection of threatened species
is released jointly by

18
www.shankariasacademy.com | www.iasparliament.com
8) Consider the following statements with c. Both 1 and 2
respect to Merchandise Export Incentive
d. Neither 1 nor 2
Scheme (MEIS)
1. Under the Scheme, the government
gives incentives to exporters‟ equivalent Answers
to a certain percentage of their export
value. 1. c
2. It is being implemented by the Ministry  The Cabinet Committee of Economic Affairs
of Commerce and Industry. recently gave its approval for the creation of a
corpus of Rs. 2000 crore for Agri-Market
Which of the statement(s) given above is/are Infrastructure Fund (AMIF) to be created
correct? with NABARD for development and up-
a. 1 only gradation of agricultural marketing
infrastructure in Gramin Agricultural
b. 2 only Markets and Regulated Wholesale Markets.
c. Both 1 and 2 2. d
d. Neither 1 nor 2  The Swachh Bharat Mission
Grameen has recently launched the Part two
of Darwaza Band campaign which focuses
9) Which of the following product(s) does on sustaining the open defecation free
not have a GI tag? status of villages across the country.
1. Rasagola  The 'Darwaza Band- Part 2' campaign has been
2. Silao Khaja supported by the World Bank and is being
rolled out countrywideimmediately after the
3. Rajamudi rice launch.
4. Tirupathi Laddu 3. c
5. Kangra Paintings  The North Atlantic Treaty Organization
Select the correct answer using the code given (NATO) is an alliance of 29 countries from
below North America and Europe committed to
fulfilling the goals of the North Atlantic Treaty
a. 3 only signed on 4 April 1949.
b. 2 and 3 only  In accordance with the Treaty, the
c. 1, 4 and 5 only fundamental role of NATO is to safeguard the
freedom and security of its member countries
d. 2, 3 and 5 only by political and military means.
 Macedonia has recently signed an accord to
10) Consider the following statements with join NATO to become the 30th member of the
respect to National Minorities Development US-led alliance.
and Finance Corporation (NMDFC)
 The Macedonia-NATO accord follows a deal
1. It is a not for profit company with Greece ending a 27-year-old dispute over
established under the companies act Macedonia's name.
1956.
 The accord must now be ratified by
2. It works under the aegis of Ministry of allied governments.
Finance.
4. b
Which of the statement(s) given above is/are
correct?  India is second only to sub-Saharan Africa in
how many poor children live in the country,
a. 1 only states a joint report on Social Protection for
b. 2 only Children released by UNICEF and

19
www.shankariasacademy.com | www.iasparliament.com
International Labour Organization  Geographical Indications Registry at Chennai
(ILO). has recently granted GI tag to Silao Khaja, the
traditional delicacy of Nalanda district, Bihar.
5. b
 Rajamudi is a traditional red rice variety of Old
 The RoSL, a scheme under which exporters
Mysore region, which was patronised by the
can claim refunds from the Centre for all the
―royals‖ (and hence the nomenclature).
levies and duties they pay at the State level,
is extended only to exporters of apparel  There is a serious effort on to secure GI tag for
and made-ups. the ‗royal rice‘.
 With India‟s eligibility to extend direct sops to  If all goes as planned, Rajamudi will join the
exporters coming under the World Trade league of Basmati and get a Geographical
Organisation (WTO) scanner, the government Indication (GI) tag in recognition of its unique
is examining the industry‟s suggestion of qualities.
expanding the scope and coverage of the
Rebate of State Levies (RoSL). 10. a

 RoSL Scheme does not flout global trade rules  The National Minorities Development and
as it involves refund of taxes and levies paid Finance Corporation (NMDFC) is a not for
by exporters, and is not a subsidy profit company under the companies act 1956.
scheme like Merchandise Export Incentive  It works under the aegis of Ministry
Scheme (MEIS). of Minority Affairs.
6. c  It is a National Level Apex Body for the benefit
7. c of Minorities as defined under the National
Commission for Minorities Act 1992.
 ‗EXERCISE RAHAT‘ is a Joint Humanitarian
Assistance and Disaster Relief Exercise.  The prime mandate of NMDFC is to provide
concessional finance to the Minorities for self-
 It was conducted by the Jaipur based Sapta employment/ income generation activities.
Shakti Command on behalf of Indian Army.
 The notified Minorities in India are Muslims,
 The Joint exercise in coordination with NDMA Christians, Sikhs, Buddhists & Parsis and Jain.
is being conducted to synergise efforts for
humanitarian assistance and disaster relief  Under NMDFC program, preference is given to
operations. Artisans & Women.
8. c
 At present, bulk of the incentives to exporters 08-02-2019
is under the popular MEIS wherein the
1) Consider the following statements with
government gives incentives to exporters‘
respect to Atal Bhujal Yojana
equivalent to a certain percentage of their
export value in the form of duty credit 1. It is a pan-Indian program aimed at
scrips that can be used to pay customs duties sustainable ground water management
and are freely transferable. through community participation.
 But with the WTO now ruling that since India‘s 2. The funding pattern is 50:50 between
per capita Gross National Income is over Government of India and World Bank.
$1,000 it is no longer eligible to give direct
Which of the statement(s) given above is/are
subsidies such as the ones offered under MEIS,
correct?
such schemes have to be phased out.
a. 1 only
 Since the government is not keen on giving
more direct export subsidies such as the one b. 2 only
given under the MEIS, the ROSL becomes
c. Both 1 and 2
more relevant.
d. Neither 1 nor 2
9. a

20
www.shankariasacademy.com | www.iasparliament.com
2) “International Intellectual Property a. The amount charged for the use of ground
Index” was recently released by which of the water
following?
b. The amount charged from the exporters of
a. World Bank water intensive crops
b. World Economic Forum c. The incentive given to consumers who follow
water conservation measures
c. World Intellectual Property Organisation
d. None of the above
d. US Chambers of Commerce

6) Consider the following statements with


3) Consider the following statements with
respect to e-NAM portal
respect to Central Bureau of Investigation
(CBI) 1. It is a pan-Indian electronic trading
portal which seeks to replace the
1. States cannot deny entry to the CBI to
existing APMC market through a virtual
probe cases referred by the
platform.
Constitutional courts.
2. This portal only supports intra-state
2. Withdrawal of consent to CBI by a state
trade and does not support inter-state
government can be effected
transactions.
prospectively and not retrospectively.
Which of the statement(s) given above is/are
Which of the statement(s) given above is/are
correct?
correct?
a. 1 only
a. 1 only
b. 2 only
b. 2 only
c. Both 1 and 2
c. Both 1 and 2
d. Neither 1 nor 2
d. Neither 1 nor 2

7) Consider the following statements with


4) Consider the following statements with
respect to Intermediate-Range Nuclear Forces
respect to Indian Ocean Rim Association
(INF) treaty
(IORA)
1. It is a treaty among UN member
1. It is an Inter-governmental
countries that prohibited development,
International organisation.
testing and possession of conventional
2. All sovereign States of the Indian Ocean as well as nuclear missiles.
Rim are eligible for membership of the
2. The treaty did not cover sea based or air
association.
launched missiles.
3. India is the current Chair of IORA.
3. Recently, U.S.A has decided to withdraw
Which of the statement(s) given above is/are from the Intermediate-Range Nuclear
correct? Forces (INF) treaty.
a. 3 only Which of the statement(s) given above is/are
correct?
b. 1 and 2 only
a. 3 only
c. 2 and 3 only
b. 1 and 2 only
d. 1, 2 and 3
c. 2 and 3 only
d. 1, 2 and 3
5) “Water Conservation Fee” sometimes seen
in the news recently is?

21
www.shankariasacademy.com | www.iasparliament.com
8) “Ekkalam” is an aerophone musical  Atal Bhujal Yojana (ABHY) aimed at
instrument mainly used in which of the sustainable ground water management
following states? with community
participation in select over-exploited and
a. Gujarat
ground water stressed areas in seven
b. Tamilnadu States (Gujarat, Haryana, Karnataka, Madhya
Pradesh, Maharashtra, Rajasthan and Uttar
c. Odisha
Pradesh).
d. West Bengal
 ABHY is designed as a Central Sector Scheme
with a total outlay of Rs. 6,000 Crore and is
proposed to be implemented with World Bank
9) Consider the following statements with
assistance.
respect to Ratoon cropping
1. It is a form of cultivation in which a  The funding pattern is 50:50 between
second crop is allowed to grow from the Government of India and World Bank.
remains of one already harvested. 2. d
2. Ratooning can be used endlessly as the  India has climbed eight places to rank 36th in
yield and quality increases after each the latest annual International IP
cycle. Index released by the US Chambers of
Which of the statement(s) given above is/are Commerce.
correct?  In 2018 International Intellectual Property
a. 1 only Index, India was ranked 44 out of 50
countries.
b. 2 only
3. c
c. Both 1 and 2
 The CBI, which functions under the provisions
d. Neither 1 nor 2 of the Delhi Special Police Establishment
(DSPE) Act, 1946, can probe offences in a state
with prior approval of the state government
10) Consider the following statements with concerned.
respect to Tornado
 Further, Constitutional courts can also
1. It is a violent, whirling windstorm that entrust any case or class of case for
moves in funnel fashion, with the broad investigation in exercise of inherent
end of the funnel scraping dark clouds jurisdiction even without the consent of
in the sky and the narrow end, touching the respective state government.
the earth.
 Further, in the cases which are referred by
2. Tornadoes occur only in the United the Constitutional courts, the entry of
States of America. CBI cannot be denied by that state as these
Which of the statement(s) given above is/are do not require the consent of the state.
correct?
 Withdrawal of consent, if any, by a state
a. 1 only government can be effected prospectively
and not retrospectively.
b. 2 only
4. b
c. Both 1 and 2
 It is an Intergovernmental
d. Neither 1 nor 2
organization which was established on 1997.
 It is a dynamic organisation of 22 Member
Answers States and 9 Dialogue Partners.

1. b

22
www.shankariasacademy.com | www.iasparliament.com
 All sovereign States of the Indian Ocean  The ban applied to missiles with both nuclear
Rim are eligible for membership of the and conventional warheads, but it did not
Association. apply to sea-based or air-delivered
missiles.
 India is a member.
 The U.S. has recently decided to withdraw
 IORA‘s apex body is the Council of Foreign from the Intermediate-Range Nuclear Forces
Ministers (COM) which meets annually. (INF) treaty with Russia.
 The Republic of South Africa assumed  The U.S.‘s unilateral withdrawal from a
the role of chair for 2017-2019. nuclear treaty threatens to trigger a new arms
 The meeting of Indian Ocean Rim Association race.
(IORA) Cluster Group on Disaster Risk 8. b
Management (DRM) to strengthen
cooperation in disaster response concluded  The ekkalam is an aerophone instrument
recently. mainly used in Tamil Nadu, India.
5. a  It is made of brass or copper. It consists of four
valves that are fastened to each other, with a
 Ground water extraction in India is regulated bell at one end.
by Central Ground Water Authority
(CGWA) constituted under  This instrument produces sound by vibration
the Environment (Protection) Act of of air injected by the player, altering the pitch
1986. by changing the lip tension and the power of
air flow.
 Central Ground Water Authority (CGWA)
under Ministry of Water Resources, River  It is played in temple festivals or celebrations.
Development and Ganga Rejuvenation had
recently notified revised guidelines to regulate  It is usually played along with a drum or with
and control ground water extraction in another ekkalam.
India, which was proposed to be 9. a
effective from 01.06.2019.
 Ratoon cropping is a form of cultivation in
 As per these revised guidelines, Water which a second crop is allowed to grow
Conservation Fee (WCF) would be levied for from the remains of one already
use of ground water depending on the harvested.
category of area, type of industry and quantum
of ground water withdrawal.  It is also called stubble cropping as the new
plants grow from the stubble of the harvested
6. d crop.
 e-NAM (National Agriculture Market) is a pan-  Sugarcane, pineapple and banana are crops in
Indian electronic trading (e-trading) portal which this method is practised.
which seeks to network the existing physical
regulated wholesale market (known as APMC  Ratooning cannot be used endlessly as the
market) through a virtual platform to create a yield and quality decrease after each cycle.
unified national market for agricultural  In sugarcane, for example, two or three
commodities. ratooning crops are possible. After which fresh
 Inter State trade of agricultural produce in planting has to be done.
wholesale mandis through National  Rice ratooning is also possible if adequate
Agriculture Market (e-NAM) platform is water is available for the second crop.
gathering pace in recent times.
10. a
7. c
 A tornado is a violent, whirling windstorm that
 Under the INF Treaty, the United States moves in funnel fashion, with the broad end of
and Soviet Union agreed to ban all land- the funnel scraping dark clouds in the sky and
based ballistic and cruise missiles with the narrow end, touching the earth.
ranges between 500 and 5,500 kilometres.

23
www.shankariasacademy.com | www.iasparliament.com
 The wind speeds are tremendous, usually d. 1, 2 and 3
about 177 kilometres per hour but sometimes
touching 480 kilometres per hour or more.
3) Consider the following statements with
 The violently rotating winds blow down almost respect to Central Asian Flyway (CAF)
everything in their path, and a powerful
updraft of air inside the funnel can suck up 1. It covers a large continental area of Asia
large objects and carry them long distances. between the Pacific and Indian
Ocean and the associated island chains.
 Tornadoes can happen practically anywhere
in the world but they occur mostly in the 2. Indian sub-continent is a part of the
United States. Central Asian Flyway (CAF).
Which of the statement(s) given above is/are
correct?
09-02-2019
a. 1 only
1) Consider the following statements with
respect to CUTLASS EXPRESS – 19 b. 2 only

1. It is a maritime exercise between Indian c. Both 1 and 2


Navy and Mauritius Navy. d. Neither 1 nor 2
2. The aim of the exercise was to progress
inter-operability for the purpose of
interdicting illegal maritime activity in 4) cVIGIL app is an initiative of which of the
the Western Indian Ocean. following agencies?

Which of the statement(s) given above is/are a. Election Commission of India


correct? b. Central Vigilance Commission
a. 1 only c. Central Board of Indirect Taxes and Customs
b. 2 only d. Food Safety and Standards Authority of India
c. Both 1 and 2
d. Neither 1 nor 2 5) Consider the following statements with
respect to Solid Fuel Ducted Ramjet (SFDR)
propulsion technology
2) Consider the following statements with
respect to Convention on Conservation of 1. It is an indigenous technology jointly
Migratory Species (CMS) developed by the ISRO and DRDO.

1. It is an environmental treaty under the 2. This technology uses the air as an


aegis of United Nations Environment oxidizer thus eliminates the need for a
Programme (UNEP). propellant.

2. India has been a Party to the CMS since Which of the statement(s) given above is/are
its inception. correct?

3. 13th Conference of Parties (COP) of the a. 1 only


Convention is going to be hosted by b. 2 only
Russia in 2020.
c. Both 1 and 2
Which of the statement(s) given above is/are
correct? d. Neither 1 nor 2

a. 1 only
b. 1 and 2 only 6) Which of the following birds/animals can be
spotted in India?
c. 2 and 3 only
1. Amur Falcons

24
www.shankariasacademy.com | www.iasparliament.com
2. Bar headed Gheese 9) Albendazole is a medication used for the
treatment of which of the following?
3. Black necked cranes
a. Seasonal Influenza
4. Marine turtles
b. Tuberculosis
5. Dugongs
c. Worm infestations
6. Humpbacked Whales
d. None of the above
Select the correct answer using the code given
below
a. 1, 3, 4 and 5 only 10) Consider the following statements with
respect to Competition Commission of India
b. 1, 2, 3 and 5 only
(CCI)
c. 2, 3, 4, 5 and 6 only
1. It is a statutory body.
d. 1, 2, 3, 4, 5 and 6
2. It is responsible for enforcing the
Competition Act, 2002 throughout
India.
7) Consider the following statements
Which of the statement(s) given above
1. HeliNa is an indigenously developed air- is/are incorrect?
to-land version of Nag missile.
a. 1 only
2. Nag missile is one of the five missile
systems developed by DRDO under the b. 2 only
Integrated Guided Missile Development
c. Both 1 and 2
Programme (IGMDP).
d. Neither 1 nor 2
Which of the statement(s) given above is/are
correct?
a. 1 only Answers
b. 2 only 1. b
c. Both 1 and 2  INS Trikand, a front-line warship of
d. Neither 1 nor 2 the Indian Navy, participated in
a multinational training
exercise ‗CUTLASS EXPRESS – 19‘ held from
8) Consider the following statements with 27 Jan to 06 Feb 19.
respect to Mera aspataal initiative  The aim of the exercise was to improve law
1. It capture patient feedback for the enforcement capacity, promote regional
services received at the hospital through security and progress inter-operability
user-friendly multiple channels. between the armed forces of the participating
nations for the purpose of interdicting illegal
2. It is an initiative launched by the maritime activity in the Western Indian
Ministry of Health and Family Welfare. Ocean.
Which of the statement(s) given above 2. b
is/are incorrect?
 The 13th Conference of Parties (COP) of
a. 1 only the Convention on the conservation of
b. 2 only migratory species of wild animals (CMS), an
environmental treaty under the aegis
c. Both 1 and 2 of United Nations Environment
d. Neither 1 nor 2 Programme, is going to be hosted by India
during 15th to

25
www.shankariasacademy.com | www.iasparliament.com
22nd February, 2020 at Gandhinagar in  ECI has also launched a Voter Verification
Gujarat. and Information Programme (VVIP) for
citizens for verifications of their names, new
 CMS, also referred to as the Bonn
registrations, and changes in the voter details
Convention provides a global platform for
and corrections in the Voter Id Cards for the
the conservation and sustainable use of
upcoming General Elections 2019 during the
migratory animals and their habitats and
workshop.
brings together the States through which
migratory animals pass, the Range States, and 5. d
lays the legal foundation for internationally
coordinated conservation measures  The SFDR technology was jointly developed
throughout a migratory range. by India (DRDO) and Russia.

 India has been a Party to the CMS since  The ramjet propulsion system used in the
1983 when the convention entered into force. SFDR acts as an oxidizer and the solid
propellant reacts as air flows through a solid
 The Conference of Parties (COP) is the propellant duct.
decision-making organ of this convention.
 Unlike conventional rockets that carry
 India has also signed non legally binding MOU propellant and oxidizer, Ramjet uses the air as
with CMS on the conservation and an oxidizer just like a jet engine. Therefore the
management of Siberian Cranes (1998), weight of the fuel required is eliminated.
Marine Turtles (2007), Dugongs (2008) and
Raptors (2016).  Defence Research and Development
Organisation (DRDO) successfully flight tested
3. b the second indigenously developed ‗Solid Fuel
Ducted Ramjet (SFDR)‘ propulsion based
 The Indian sub-continent is also part of
missile system from ITR, Chandipur, Odisha
the major bird flyway network, i.e,
recently.
the Central Asian Flyway (CAF) that
covers areas between the Arctic and Indian  The success of SFDR propulsion technology
Oceans, and covers at least 279 populations will pave the way for the development of long-
of 182 migratory water bird species, including range air-to-air missiles in the country.
29 globally threatened species.
6. d
 India has also launched the National Action
Plan for conservation of migratory  India is temporary home to several migratory
species under the Central Asian Flyway. animals and birds.

4. a  The important among these include Amur


Falcons, Bar headed Gheese, Black
 Election Commission of India (ECI) has necked cranes, Marine turtles,
recently conducted a 2-day intensive Training- Dugongs, Humpbacked Whales, etc.
cum-Workshop on the use of Information &
Communication Technology for the conduct of 7. c
General Elections 2019.  HeliNa is an air-to-land version of Nag
 One of the important applications for which missile, which is one of the five missile
training was imparted to all States and UTs systems developed by DRDO under the
was cVIGIL. Integrated Guided Missile Development
Programme (IGMDP).
 cVIGIL App provides time-stamped, evidence
based proof of Model Code of Conduct /  India, successfully flight tested
Expenditure Violation, having live photo/video the indigenously developed and modified
with auto location data. anti-tank missile HeliNa, guided from a
defence facility off Odisha coast paving the way
 Any citizen can lodge a complaint through for its actual launch from helicopter.
Mobile App.
8. d
 Flying Squads then investigate the matter and
the Returning Officer takes the decision.

26
www.shankariasacademy.com | www.iasparliament.com
 Mera aspataal (My Hospital) is a Ministry of b. 1, 2 and 4 only
Health, Government of India initiative to
c. 1, 3 and 4 only
capture patient feedback for the
services received at the hospital through d. All of the above
user-friendly multiple channels such as Short
Message Service (SMS), Outbound Dialling
(OBD) mobile application and web portal. 3) Consider the following statements regarding
“Hydro Seismicity”,
 The patient can submit the feedback in seven
different languages on mobile app and web a. It is a phenomenon of occurrence of multiple
portal; for the hospitals visited in last 7 days. tremors under water caused by the slow
movement of tectonic plates.
 It will help the government to take appropriate
decisions for enhancing the quality of b. It is caused by the emergence of geothermally
healthcare delivery across public facilities heated groundwater that rises from the Earth‘s
which will improve the patient‘s experience. crust
9. c c. It refers to rise in groundwater pressure
released in earthquake swarms due to entering
10. b of heavy rainfall in small fractures in rocks
 Competition Commission of India is d. It refers to an induced earthquake caused by
a statutory body of the Government of hydraulic fracturing.
India, responsible for enforcing
the Competition Act, 2002 throughout
India (except J&K) and to prevent activities 4) Open Acreages Licensing Policy is related to
that have an adverse effect on competition.
a. Exploration of Hydrocarbons
b. Land acquisition under UDAN
11-02-2019
c. National Highway Development
1) One Astronomical unit correspond to the
distance between d. Swacch Bharat

a. Sun and Moon


b. Moon and Earth 5) Consider the following statements about
Akshaya Patra Foundations
c. Earth and Sun
1. It is a non-profit organisation
d. None of the above
2. It is an implementing partner of Mid-
Day Meal Scheme
2) Which of the following actions are carried Which of the above statements is/are true?
out by Food and Agriculture Organization
(FAO)? a. 1 only

1. To help eliminate hunger, food b. 2 only


insecurity and malnutrition c. Both 1 & 2
2. Make agriculture, forestry and fisheries d. Neither 1 nor 2
more productive and sustainable
3. Reduce rural poverty
6) Which of the following statements about
4. Increase the resilience of livelihoods to “Kaleswaram Project” is true?
threats and crises
a. It is a part of Mission Bhagiratha implemented
Select the correct answer using the code given by Telangana to provide piped drinking water
below to every household
a. 1, 2 and 3 only

27
www.shankariasacademy.com | www.iasparliament.com
b. It is an irrigation project implemented by 2. The discussions that happen in the
Maharashtra to provide water supply in forum are non-binding
drought hit Vidharbha region
3. Ulaanbaatar declaration was aimed to
c. It is an irrigation project implemented in prevent drug trafficking.
Telangana to utilize water from Godavari River
Which of the above statement(s) is/are
d. It is a part of Polavaram multi-purpose correct?
irrigation project implemented by Andhra
a. 1 and 2 only
Pradesh.
b. 2 and 3 only
c. 1 and 3 only
7) Consider the following statements
regarding “Sampoorna Bima Gram Yojana” d. All of the above
1. It is implemented by Ministry of
Communication.
10) Consider the following statements
2. Its objective is to provide internet
connectivity to villages by establishing 1. Algae can be used to generate bio-fuels
optical fibre network 2. Fungi are natural bio-indicators
Which of the above statement(s) is/are 3. Anthrax is caused by bacteria.
correct?
Which of the above statement(s) is/are
a. 1 only correct?
b. 2 only a. 1 and 2 only
c. Both 1 & 2 b. 1 and 3 only
d. Neither 1 nor 2 c. 2 and 3 only
d. All of the above
8) Consider the following statements regarding
Kanger Ghati National Park
Answers
1. It is located near Jagdalpur in the
Bastar region of Chhatisgarh. 1. c
2. It is known for Hill Myna.  As the distance between the Earth and the Sun
varies during Earth‘s revolution and with
3. It is located in the catchment area of improving technology affording better
Godavari River. measurements, the definition of an AU by the
Which of the above statement(s) is/are International Astronomical Union (IAU) has
correct? varied with time.
a. 1 only  The latest definition was adopted in a meeting
of the IAU in 2012, when members voted to
b. 1 and 3 only make the AU an exact 149,597,870,700 mt,
c. 2 and 3 only which is the average mean distance between
the Earth and the Sun when viewed from the
d. All of the above Earth.
2. d
9) Consider the following statements 3. c
about Asia–Europe Meeting (ASEM)
 In ―hydro-seismicity‖ water from heavy rainfall
1. It is an exclusively Asian–European enters small fractures in rocks, which raises
forum. the pressure.

28
www.shankariasacademy.com | www.iasparliament.com
 The pressure built up due to the rise in ground Which of the statement(s) given above is/are
water is released in earthquake swarms. correct?
 It is estimates that with every 10 meter rise in a. 1 only
groundwater pore pressure increases by 1 bar. b. 2 only
4. a c. Both 1 and 2
5. c d. Neither 1 nor 2
6. c
7. a 3) Consider the following statements with
 Its objective is to give rural people affordable respect to Swachh Shakti-2019
life insurances services. 1. It is a national event attended by women
8. d Panches and Sarpanches from across
the country.
9. a
2. The First edition of the event Swachh
 Ulaanbaatar declaration resolved to fight Shakthi will be held in Kurukshetra,
against terrorism. Haryana.
10. d 3. It is being organized by the Ministry of
Women and Child Development in
association with the Govt. of Haryana.
12-02-2019 Which of the statement(s) given above is/are
1) Consider the following statements with correct?
respect to Gateway of India a. 1 only
1. It is an arch monument built during b. 1 and 2 only
20th century.
c. 2 and 3 only
2. It was erected to commemorate the
Indian soldiers who lost their lives d. 1, 2 and 3
fighting for the British Army during the
World War I.
4) Blue Flame Revolution sometimes seen in
3. It is built in Indo-Saracenic style.
the news recently, is associated with which of
Which of the statement(s) given above is/are the following?
correct?
a. Nuclear Energy
a. 1 only
b. Clean cooking fuel
b. 1 and 2 only
c. Fishing and its allied sectors
c. 1 and 3 only
d. Offshore oil production
d. 1, 2 and 3

5) Consider the following statements with


2) Consider the following statements with respect to Cultural Heritage Youth Leadership
respect to World Sustainable Development Programme (CHYLP)
Summit (WSDS)
1. It envisaged enriching awareness of
1. It is the annual flagship event organised Indian culture amongst the youth in
by NITI Aayog. order to promote fondness for India‟s
rich cultural heritage.
2. WSDS 2019 will focus on the theme,
'Attaining the 2030 Agenda: Delivering 2. Currently, it is being implemented by
on our Promise'. the National Council of Educational
Research and Training (NCERT).

29
www.shankariasacademy.com | www.iasparliament.com
Which of the statement(s) given above is/are 9) Dard Aryan tribes sometimes seen in the
correct? news recently, inhabit which of the following
states?
a. 1 only
a. Jammu and Kashmir
b. 2 only
b. Odisha
c. Both 1 and 2
c. Chattisgarh
d. Neither 1 nor 2
d. Arunachal Pradesh

6) DD Arunprabha, a dedicated 24x7


Doordarshan satellite channel was launched 10) Consider the following statements
recently for which of the following states?
1. India is the second largest producer of
a. Jammu and Kashmir silk after China.
b. Arunachal Pradesh 2. India is the largest consumer of silk.
c. Chhattisgarh Which of the statement(s) given above is/are
correct?
d. Uttarakhand
a. 1 only
b. 2 only
7) Every Finance Commission has a special
theme. What is the special theme of the c. Both 1 and 2
Fifteenth Finance Commission?
d. Neither 1 nor 2
a. Health and Education
b. GST Restructuring
Answers
c. Poverty and Unemployment
1. c
d. Global warming and Climate Change
 The Maharashtra government has recently
initiated a plan to clean, restore and beautify
8) Consider the following statements with the iconic Gateway of India in South Mumbai.
respect to e-office Project  The Gateway of India is an arch monument
1. It is a mission mode project under the built during the 20th century.
National e-Governance Plan.
 The iconic structure, which overlooks the
2. It seeks to replace the physical files and Arabian Sea, was erected to commemorate the
documents with an efficient electronic visit of King George V and Queen Mary at
system. Apollo Bunder to Mumbai.
3. It is being implemented by the  The foundation stone of the monument was
Department of Administrative Reforms laid on March 31, 1913 and its construction was
and Public Grievances (DARPG). completed in 1924.
Which of the statement(s) given above is/are  It is built in Indo-Saracenic style.
correct?
 India Gate which stands at the centre of New
a. 1 only Delhi commemorates the 70,000 Indian
b. 1 and 3 only soldiers who lost their lives fighting for the
British Army during the World War I.
c. 2 and 3 only
 The memorial bears the names of more than
d. 1, 2 and 3 13,516 British and Indian soldiers killed in the
North-western Frontier in the Afghan war of
1919.

30
www.shankariasacademy.com | www.iasparliament.com
2. b envisaged enriching awareness of Indian
cultureand heritage amongst the youth in
 The World Sustainable Development Summit order to promote, understand and develop
(WSDS) is the annual flagship event of The fondness for India‘s rich cultural
Energy and Resources Institute(TERI). heritage, with a view to develop
 WSDS 2019 will focus on the theme, appropriate leadership qualities
'Attaining the 2030 Agenda: Delivering amongst youth.
on our Promise'.  The focus of the programme was to be on less
 The ―mega themes‖ to be addressed during the privileged children residing in backward
event are clean oceans, climate finance, energy areas by interacting with them
transitions, mobility and sustainable in vernacular languages for their better
agriculture. understanding.
3. a  Intermittently programme was given
to Centre for Cultural Resources and
 Swachh Shakti-2019 is a national Training (CCRT), an autonomous
event which aims to bring in to focus the organization working under the purview of
leadership role played by rural women in Ministry of Culture.
Swachh Bharat Mission.
6. b
 Women Sarpanches and Panches from all
over the country will be attending the  A New dedicated DD Channel, for Arunachal
event. Pradesh - DD Arun Prabha was launched
by PM at IG Park, Itanagar.
 The Ministry of Drinking Water and
Sanitation in association with the Govt. of  The channel will be the 24th channel operated
Haryana is organizing the Swachh Shakti- by Doordarshan.
2019.  Through this channel news of even remote
 Best practices from grass root level in the rural places in the state will reach the people.
areas for Swachh Bharat will be shared by 7. a
them.
 Every Finance Commission has a special
 The event will showcase the achievements of theme.
Swachh Bharat and the recently conducted
Swachh Sunder Shauchalay, (neat and clean  ‗GST Restructuring‘ was the special theme of
toilet) - a unique and first of its kind in the the Thirteenth Finance Commission headed by
world campaign. Shri Vijay L. Kelkar.
4. b  Health and Education would be a special
theme of the Fifteenth Finance
 Recently, Prime Minister of India has declared Commission headed by Shri Nand Kishore
that a ‗Blue Flame Revolution‘ is under-way. Singh.
 LPG coverage has reached more than 90% 8. d
percent, from 55% five years ago.
 The e-Office is a Mission Mode Project
 The Union Minister for petroleum and Natural under the National e-Governance
gas has recently stated that, ―In addition to the Plan and is being implemented by
general development of the hydrocarbon the Department of Administrative
industry, we have also focused on improving Reforms and Public Grievances (DARPG)
the ease of living of the common people by to improve efficiency in Government Process
enhancing access to clean cooking fuel which and Service Delivery Mechanism.
we have called the Blue Flame Revolution‖.
 The e-Office is a Digital Workplace Solution
5. a that replaces the physical files and
 The scheme for Cultural Heritage Youth documents with an efficient electronic
Leadership Programme (CHYLP) system.

31
www.shankariasacademy.com | www.iasparliament.com
9. a 1. Cholesterol is a waxy, fat-like substance
produced in the liver.
 Dard Aryans Tribes inhabit Dha, Hanu,
Beema, Darchik and Garkone villages in Leh 2. High density Lipoprotein (HDL)
and Kargil districts and are together called cholesterol and Low density Lipoprotein
the Aryan valley. (LDL) cholesterol are the two main
types of cholesterol.
 The people of this region have unique physical
features, social life, ethnic culture and 3. High HDL levels reduce the risk for
language. heart disease.

 Researchers believe that the ‗Aryans of Ladakh‘ Which of the statement(s) given above is/are
or the ‗Brokpas‘ are descendants of Alexander‘s correct?
army and had come to the region over 2,000 a. 2 only
years ago.
b. 1 and 2 only
 A delegation of the Dard Aryans recently
submitted their charter of demands to c. 2 and 3 only
Minister of State for Tribal Affairs. d. 1, 2 and 3
10. c

4) Consider the following statements with


13-02-2019 respect to “PCSK9 protein”
1. PCSK9 protein reduces the low density
1) Consider the following statements with
lipoprotein (LDL) cholesterol in blood.
respect to Tangkhul Naga tribes
2. PCSK9 inhibitors are monoclonal
1. Tangkhul Naga is an indigenous tribal
antibodies (MABs) which dramatically
community inhabiting ukhrul district of
increase LDL cholesterol levels by fight
Manipur.
against the PCSK9 protein.
2. Luira Phanit is a harvesting festival
Which of the statement(s) given above is/are
celebrated by these tribes.
correct?
Which of the statement(s) given above is/are
a. 1 only
correct?
b. 2 only
a. 1 only
c. Both 1 and 2
b. 2 only
d. Neither 1 nor 2
c. Both 1 and 2
d. Neither 1 nor 2
5) A report was released recently titled: “India
– Spearheading Climate Solutions”. Consider
2) Zoologists have recently discovered a new the following statements in this regard
species called “Crying Snake” in which of the
1. The report comprises of guidelines that
following?
have to be followed by all stakeholders
a. Kerala to combat and adapt to climate change.
b. Rajasthan 2. It was released by the Ministry of
Environment, Forest and Climate
c. Arunachal Pradesh
change.
d. Andaman & Nicobar Islands
Which of the statement(s) given above is/are
correct?
3) Consider the following statements with a. 1 only
respect to Cholesterol
b. 2 only

32
www.shankariasacademy.com | www.iasparliament.com
c. Both 1 and 2 d. 1, 2, 3 and 4
d. Neither 1 nor 2
9) “AMAN-19” sometimes seen in the news
recently, is a multinational maritime exercise
6) “e-Cocoon” Application was recently
hosted by which of the following countries?
launched by which of the following?
a. Pakistan
a. Ministry of Textiles
b. Saudi Arabia
b. Ministry of Commerce and Industry
c. India
c. Ministry of Agriculture and Farmers welfare
d. Jointly by A and B
d. Agricultural Products Export Development
Authority
10) Match the following
7) Consider the following statements with 1. Sela pass – a. Uttarakhand
respect to Small Grants Program (SGP)
2. Khardung La – b. Arunachal
1. It provides both financial and technical Pradesh
support to communities and Civil
3. Sin La – c. Jammu and
Society Organizations to meet the
Kashmir
overall objective of global
environmental benefits secured through Select the correct answer using the code given
community-based initiatives and below
actions.
a. 1-a; 2-b; 3-c
2. It is implemented by UNDP on behalf of
the GEF (Global Environment Facility) b. 1-c; 2-a; 3-b
partnership. c. 1-a; 2-c; 3-b
Which of the statement(s) given above is/are d. 1-b; 2-c; 3-a
correct?
a. 1 only
Answers
b. 2 only
1. d
c. Both 1 and 2
 The Tangkhul-Naga tribe came to Manipur,
d. Neither 1 nor 2 Nagaland, Assam and Arunachal
Pradesh from China through Myanmar,
entering their present habitats in successive
8) In which of the following areas, space waves of immigration from 9th to 11th century
technology can be used? BC.
1. Island development and security  Most of them live in Manipur's Ukhrul
2. Border Surveillance district in North East India.
3. Communication and Navigation  Luira phanit is a seed sowing festival
celebrated by the Tangkhul Naga tribes.
4. GIS and Operations Planning System
2. c
Select the correct answer using the code given
below  A new species of „crying‟ snake has been
discovered in Lepa-Rada district of
a. 2 and 3 only
Arunachal Pradesh.
b. 2 and 4 only
 ―The name for this keelback was suggested
c. 1, 2 and 3 only because of a dark spot under its eyes looking

33
www.shankariasacademy.com | www.iasparliament.com
like a black tear that interrupts a white stripe 6. a
running along the upper jaw to the back of its
head and beyond.  Ministry of Textiles has launched a mobile
application for quality certification in silkworm
 The discovery of the non-venomous crying seed sector.
keelback, whose zoological name is Hebius
lacrima, has been published in Zootaxa, the  The app will be used by the Seed Analysts and
New Zealand-based scientific mega-journal Seed Officers nominated under Central Seed
for animal taxonomy. Act for system and product certification
through real time reporting.
3. d
7. c
 Cholesterol is a waxy, fat-like substance
produced in the liver.  Small Grants Program (SGP) is implemented
by UNDP on behalf of the GEF (Global
 High density Lipoprotein (HDL) cholesterol Environment Facility) partnership.
and Low density Lipoprotein (LDL) cholesterol
are the two main types of cholesterol.  SGP provides financial and technical
support to communities and Civil Society
 LDL cholesterol is considered the ―bad‖ Organizations to meet the overall objective of
cholesterol, because it contributes to fatty global environmental benefits secured through
build-ups in arteries (atherosclerosis). community-based initiatives and actions.
 This condition narrows the arteries and  Through a decentralized, national-level
increases the risk for heart attack, stroke and delivery mechanism, SGP finances
peripheral artery disease, or PAD. community-led initiatives to address global
environmental issues.
 HDL cholesterol can be thought of as the
―good‖ cholesterol. (So, in the case of HDL  The Program is specifically designed to
cholesterol, higher levels are actually better). mobilize bottom-up actions by empowering
local civil society organizations, and poor and
 Experts believe that HDL acts as a scavenger, vulnerable communities, including women and
carrying LDL (bad) cholesterol away from the Indigenous Peoples.
arteries and back to the liver, where the LDL is
broken down and passed from the body. 8. d
4. d  Union Government has formed a Task Force
for identifying areas for use of space
 Proprotein convertase subtilisin/kexin type 9 technology in improving border management.
(PCSK9) binds to low-density lipoprotein
receptors (LDL receptors), which stops LDL  Details of the areas identified for use of space
being removed from the blood, leading to technology by the task force are as under:-
an increase in blood levels of LDL.
1. Island development and security
 The PCSK9 inhibitor blocks the PCSK9,
2. Border Surveillance
resulting in more LDL receptors available
to remove LDL from the blood, which 3. Communication and Navigation
produces in a decrease in LDL blood levels.
4. GIS and Operations Planning System
5. b
5. Border Infrastructure Monitoring
 Union Ministry of Environment, Forest and 9. a
Climate Change released a publication recently
in New Delhi on climate actions in India titled  A five-day multinational exercise hosted
―India – Spearheading Climate by Pakistan‘s navy has kicked off near the
Solutions‖. southern port city of Karachi as part of efforts
to enhance cooperation combating pirates,
 This publication mentions the key terrorists and smugglers.
actions India has taken under various
sectors towards combating and adapting to  Representatives from the navies of 45 nations,
climate change. including the United States, Britain and Japan,

34
www.shankariasacademy.com | www.iasparliament.com
are participating in the exercise titled ―Aman‖ 2) e-AUSHADHI portal which intended for
or peace, which began Friday in the Northern increased transparency was launched recently
Arabian Sea. by which of the following agencies?
 Pakistan has been hosting the exercise since a. Ministry of AYUSH
2007 but has never invited arch rival
b. Ministry of Health and Family Welfare
neighbouring India.
c. Ministry of Chemicals and Fertilizers
10. d
d. Government of Haryana
 The foundation stone for Sela Tunnel Project
was laid recently.
 Sela Pass also called as Se La is the high- 3) Consider the following statements with
altitude mountain pass located in Tawang respect to National Security Act (NSA)
District of Arunachal Pradesh. 1. It allows preventive detention for
 It has an elevation of 4170 m (13,700 ft) months, if authorities are satisfied that
connects the Buddhist city of Tawang Town to a person is a threat to national security
Tezpur and Guwahati. or law and order.

 It is the main road connecting Tawang with the 2. It applies to the entirety of India, except
rest of India. Jammu and Kashmir.

 The Sela Lake also known as Paradise Lake is Which of the statement(s) given above is/are
located near to the pass. correct?
a. 1 only
 It is usually open throughout the year unless
landslides or snow require the pass to be shut b. 2 only
down temporarily.
c. Both 1 and 2
d. Neither 1 nor 2
14-02-2019
1) Consider the following statements with 4) Consider the following statements with
respect to National Commission for Safai respect to Battle of Sinhagad
Karamcharis (NCSK)
1. It was fought between the Marathas and
1. It is a statutory body. the British on the fort of Sinhagad in
2. It works under the Ministry of Social 1670 AD.
Justice and Empowerment. 2. Tanaji Malusare, the commander of
3. NCSK is mandated to monitor the Mughal Empire played a major role in
implementation of the Prohibition of this battle and died after being seriously
employment as manual scavengers and wounded during the battle.
their rehabilitation Act, 2013. Which of the statement(s) given above is/are
Which of the statement(s) given above is/are correct?
correct? a. 1 only
a. 1 only b. 2 only
b. 1 and 3 only c. Both 1 and 2
c. 2 and 3 only d. Neither 1 nor 2
d. 1, 2 and 3

35
www.shankariasacademy.com | www.iasparliament.com
5) Consider the following statements with 2. It is mandated to mobilize farmers into
respect to Taj Trapezium Zone (TTZ) groups, called Farmers Producers
Organisations (FPOs) and link these
1. It is a defined area around the Taj
institutions to the market, for better
Mahal to protect the monument from
returns.
pollution.
Which of the statement(s) given above is/are
2. It comprises over 40 protected
correct?
monuments including three World
Heritage Sites: Agra Fort, India Gate a. 1 only
and the Fatehpur Sikri.
b. 2 only
3. TTZ is so named since it is located
c. Both 1 and 2
around the Taj Mahal and is shaped like
a trapezoid. d. Neither 1 nor 2
Which of the statement(s) given above is/are
correct?
9) Consider the following statements with
a. 1 only respect to National Board for Wildlife (NBWL)
b. 1 and 2 only 1. The standing committee of the NBWL is
chaired by the Union minister
c. 1 and 3 only
for Environment and Forest.
d. 1, 2 and 3
2. In India, NBWL is responsible for
allowing forest land in protected areas
to be diverted for industries.
6) Global Fund sometimes seen in the news
recently, is a partnership organization, Which of the statement(s) given above is/are
designed to accelerate the end of which of the correct?
following as epidemics?
a. 1 only
a. AIDS
b. 2 only
b. Tuberculosis
c. Both 1 and 2
c. Malaria
d. Neither 1 nor 2
d. All of the above

10) “Pashtuns” sometimes seen in the news


7) “Factsheet Child Marriages 2019” is a report recently, inhabits which of the following
published recently by which of the following countries?
agencies?
a. Syria
a. UNICEF
b. Israel
b. Bachpan Bachao Andolan
c. Azerbaijan
c. National Human Rights Commission
d. None of the above
d. Ministry of Women and Child Development

Answers
8) Consider the following statements with
respect to Small Farmers Agribusiness 1. c
Consortium (SFAC)  The National Commission for Safai
1. It is an autonomous society working Karamcharis (NCSK) was established as a
under the administrative control of statutory body in the year 1994 as per the
Ministry of Agriculture & Farmers provisions of the NCSK Act 1993 initially for
Welfare. the period of 3 years.

36
www.shankariasacademy.com | www.iasparliament.com
 Later the validity of the Act was initially  Tanaji Malusare, the brave Maratha warrior
extended up to 31.3.2002 and thereafter up to played a major role during the Battle of
29.2.2004. Sinhagad in 1670 AD.
 The NCSK Act ceased to have effect from  Malusare died in 1670 after being seriously
29.2.2004. wounded in the Sinhagad battle.
 After that the tenure of the NCSK has been  It is Tanaji Malusare after whom Shivaji is said
extended as a non-statutory body from to have renamed Kondhana fort to Sinhagad
time to time. Fort (Lion‘s fort).
 The tenure of the present Commission is up to 5. c
31.3.2019.
 Taj Trapezium Zone (TTZ) is a defined area of
 The Union Cabinet has recently approved the 10,400 sq km around the Taj Mahal to protect
proposal for Extension of tenure of the the monument from pollution.
National Commission for Safai Karmacharis
 The TTZ comprises over 40 protected
(NCSK) beyond 31.3.2019 for three years.
monuments including three World Heritage
2. a Sites the Taj Mahal, Agra Fort and Fatehpur
Sikri.
 To increase transparency and accountability,
the Ministry of AYUSH has recently  TTZ is so named since it is located around the
launched the e-AUSHADHI Taj Mahal and is shaped like a trapezoid.
portal for online licensing of Ayurveda,
6. d
Siddha, Unani and Homoeopathy drugs and
related matters.  The Global Fund is a 21st-century partnership
organization designed to accelerate the end
 AUSHADHI was an acronym for Ayurveda,
of AIDS, tuberculosis and malariaas
Unani, Siddha and Homeopathy
epidemics.
Automated Drug Help Initiative.
3. c  Founded in 2002, the Global Fund is a
partnership between governments, civil
 The National Security Act is a stringent law society, the private sector and people
that allows preventive detention for affected by the diseases.
months, if authorities are satisfied that a
person is a threat to national security or law  India is in a sustained partnership with
and order. the Global Fund to fight against AIDS,
Tuberculosis and Malaria (GFATM) since
 The person does not need to be charged during 2002 as recipient as well as donor.
this period of detention.
 The Global Fund raises funds in multiyear
 The goal is to prevent the individual from cycles known as Replenishments and is
committing a crime. preparing for sixth voluntary replenishment
conference to mobilize needed resources to
 It applies to the entirety of India, except scale up life-saving programs over 2020-22.
Jammu and Kashmir.
 A high level preparatory meeting for the sixth
4. d
replenishment conference of the Global
 The Battle of Sinhagad was fought between Fund is hosted by India recently in New
Tanaji Malusare, a commander Delhi.
of Maratha ruler Shivaji Maharaj and 7. a
Udaybhan Rathod, fort keeper under Jai Singh
I who was a Mughal Army Chief in 1670 on  United Nations International Children's
the fort of Sinhagad near the city of Pune, Emergency Fund (UNICEF) has recently
Maharashtra, India. released a new report 'Factsheet Child
Marriages 2019'.

37
www.shankariasacademy.com | www.iasparliament.com
 It has stated that although child marriages in  Pashtun people are Afghanistan's largest
India have declined, a few states like Bihar, ethnic group and are also the second-largest
West Bengal and Rajasthan continue to carry ethnicity in Pakistan.
on with the harmful practice and there is
nearly 40 per cent prevalence in these states.  Pashtuns are united by the Pashto language,
which is a member of the Indo-
8. c Iranian language family, although many also
speak Dari (Persian) or Urdu.
 SFAC is a registered society set up under the
Societies Registration Act XXI of 1860 under  They are also known as "Pathans".
the administrative control of Department of
Agriculture Cooperation and Farmers
Welfare, Ministry of Agriculture & 15-02-2019
Farmers Welfare.
1) Consider the following statements with
 Its mandate is to link farmers to investment, respect to Pradhan Mantri Shram Yogi Maan-
technology and markets. Dhan (PM- SYM) Scheme
 SFAC is mandated by the M/o Agriculture to 1. Under the Scheme, the beneficiary
mobilize farmers into groups, called Farmers should not be an income tax payer.
Producers Organisations (FPOs) and link these
institutions to the market, for better returns. 2. It is implemented through Life
Insurance Corporation of India (LIC).
 These bodies are member owner grass root
institutions, which provide collective Which of the statement(s) given above is/are
bargaining powers to small and marginal correct?
farmers. a. 1 only
 SFAC proposes to establish a Kisan Mandi – b. 2 only
as an additional marketing channel/platform
to link FPOs and farmers Growers c. Both 1 and 2
Association to wholesale and retail consumers d. Neither 1 nor 2
for direct sale of fruits & vegetables for the
benefits of farmers and consumers in Delhi &
NCR. 2) Anoop Satpathy committee sometimes seen
9. c in the news recently, was constituted to review
and recommend which of the following?
 National Board for Wildlife (NBWL) is a
statutory Board constituted in 2003 under a. Methodology for fixation of National
Section 5 of the Wild Life (Protection) Act, Minimum Wage
1972. b. Measures to reorient disinvestment policy of
India
 The NBWL is chaired by the Hon‘ble Prime
Minister. c. Suggestions to avoid future bank frauds in
India
 The Standing Committee of the NBWL is
chaired by Union Environment Minister. d. Guidelines to combat reducing glacier cover
over Himalayas
 A Standing Committee of the NBWL is charged
with deliberating on the merits of projects that
come to it for scrutiny. 3) Consider the following statements with
 National Board for Wildlife (NBWL) has been respect to Nord Stream-2 Project
charged with allowing forest land in Protected 1. It is an expansion of Russia‟s existing
Areas to be diverted for industry. Nord Stream gas pipeline to Germany
10. d across the Caspian Sea.

 Pashtuns mainly live in Pakistan and 2. U.S.A is opposing the project citing the
Afghanistan. increase in Russian gas supply to

38
www.shankariasacademy.com | www.iasparliament.com
Europe would increase Moscow‟s Which of the statement(s) given above is/are
leverage over European countries. correct?
Which of the statement(s) given above a. 1 only
is/are incorrect?
b. 2 only
a. 1 only
c. Both 1 and 2
b. 2 only
d. Neither 1 nor 2
c. Both 1 and 2
d. Neither 1 nor 2
8) Kattaikkuttu is a rural theatre form,
practiced in which of the following states?
4) “World Employment and Social Outlook” is a. Tamil Nadu
an annual report published by which of the
b. Telangana
following organisations?
c. Odisha
a. World Bank
d. Rajasthan
b. International Labour Organisation
c. International Organisation of Employers
9) Which of the following is/are not the
d. Oxfam International
features of Indus Valley Civilisation (IVC)
1. Use of Pottery wheels
5) Green New Deal (GND) sometimes seen in
2. Undeciphered script
the news recently, is associated with which of
the following countries? 3. Proper town planning
a. U.S.A 4. Domestication of animals
b. France 5. Use of standard weights and
measurements
c. Germany
Select the correct answer using the code given
d. U.K
below
a. 4 only
6) Spirit and Opportunity rovers sometimes
b. 4 and 5 only
seen in the news recently, are associated with
which of the following? c. 1, 2, 3, 4 and 5
a. NASA and MOON d. None
b. ESA and JUPITER
c. JAXA and RYUGU 10) Consider the following pairs
d. NASA and MARS 1. Pahari Dam – Madhya Pradesh
2. Kawal Tiger Reserve – Andhra Pradesh
7) Consider the following statements with Which of the pair(s) given above is/are
respect to Sambar deer correctly matched?
1. It is large in size and endemic to Indian a. 1 only
subcontinent.
b. 2 only
2. It has been protected under schedule I
of the Wildlife Protection Act, 1972. c. Both 1 and 2
d. Neither 1 nor 2

39
www.shankariasacademy.com | www.iasparliament.com
Answers  It would also make Russia richer, and as a
result, more aggressive.
1. c
 PM-SYM is a voluntary and contributory
pension scheme on a 50:50 basis (contribution
shall be made by the beneficiary and the
matching contribution by the Central
Government).
 The unorganised workers whose monthly
income is Rs 15,000/ per month or less and
belong to the entry age group of 18-40 years
are eligible for the scheme.
 They should not be covered under New
Pension Scheme (NPS), Employees‘ State

Insurance Corporation (ESIC) scheme or
Employees‘ Provident Fund Organisation 4. b
(EPFO).
 World Employment and Social
 Further, he/she should not be an income Outlook is an annual report published
tax payer. by International Labour Organisation
(ILO), a UN agency.
 Each subscriber under the PM-SYM, shall
receive minimum assured pension of Rs  Global unemployment levels fell one percentile
3000/- per month after attaining the age of 60 to a steady 5 per cent in 2018, the lowest
years. figures since the economic crisis that wreaked
havoc on labour markets, the International
 It will be a Central Sector Labour Organization said in its
Scheme administered by the Ministry of recent report World Employment and
Labour and Employment Social Outlook Trends 2019.
and implemented through Life Insurance
Corporation of India and CSCs. 5. a

 LIC will be the Pension Fund Manager and  U.S has recently proposed the Green New
responsible for Pension pay out. Deal (GND) to address both climate change
and economic inequality.
2. a
 The Green New Deal ―is a four-part
 The Ministry of Labour and Employment had programme for moving America quickly out of
constituted an expert committee, under the crisis into a secure, sustainable future.
Chairmanship Dr. Anoop Satpathyto review
and recommend methodology for fixation  The resolution was introduced in the House
of National Minimum Wage (NMW). and Senate recently.
 The Expert Committee has submitted its 6. d
report recently.  Spirit and Opportunity were identical,
3. a golf-cart-sized, solar-powered rovers.
 Nord Stream 2 is an expansion of Russia‘s  Spirit landed at Gusev Crater on January 4,
existing Nord Stream gas pipeline to Germany 2004; Opportunity followed, landing on the
across the Baltic Sea. opposite side of Mars at Meridiani Planum on
January 24.
 The U.S says the new pipeline, and the
resultant increase in Russian gas supply to  Contact with Spirit was lost in March 2010,
Europe, would increase Moscow‘s leverage and the mission was declared over on May 25,
over European Countries. 2011.

40
www.shankariasacademy.com | www.iasparliament.com
 Opportunity continued to roam Mars — and b. National Commission on Agriculture
sent out its last signals on June 10, 2018, when
c. Ministry of Women and Child Development
the most intense dust storm in recorded
Martian history encrusted its solar panels, and d. Delhi Police Department
damaged critical components.
 NASA has recently announced the end of 3) Consider the following statements with
the Opportunity rover‟s mission. respect to Right to Information Act, 2005
7. d 1. It mandates that, a Chief Information
 The sambar deer is a large deer native to the Commissioner should be a person
Indian subcontinent, southern China, belonging to Public service, of any
and Southeast Asia. Government for more than 20 years.

 It has been listed as Vulnerable on the IUCN 2. Matters related to Collegium and
Red List. judicial appointments of Supreme Court
comes within the ambit of Right to
 Sambar is a protected animal under Information Act, 2005.
schedule three of Wildlife Protection Act,
1972. Which of the statement(s) given above is/are
correct?
8. a
a. 1 only
9. d
b. 2 only
10. d
c. Both 1 and 2
d. Neither 1 nor 2
16-02-2019
1) Consider the following statements with
4) Consider the following statements with
respect to Most Favoured Nation (MFN) status
respect to Exercise Vayu Shakti-2019
1. It is a non-discriminatory trade policy to
1. It is an air exercise between India and
discourage exclusive trading privileges
France.
among WTO countries.
2. It will be conducted once in three years.
2. India had recently revoked the Most
Favoured Nation (MFN) status bestowed Which of the statement(s) given above is/are
on Pakistan in 1996. correct?
3. Pakistan had revoked the MFN status a. 1 only
granted to India, after the Pakistan
b. 2 only
School massacre in 2014.
c. Both 1 and 2
Which of the statement(s) given above is/are
correct? d. Neither 1 nor 2
a. 2 only
b. 1 and 2 only 5) In which of the following conditions, corals
cannot survive?
c. 1, 2 and 3
1. No Sunlight
d. None
2. Fresh water
3. Estuaries
2) LADIS Portal sometimes seen in the news
recently, was launched by which of the 4. Cold water
following agencies?
Select the correct answer using the code given
a. Inland Waterways Authority of India below

41
www.shankariasacademy.com | www.iasparliament.com
a. 2 and 3 only c. 1, 2 and 3
b. 2 and 4 only d. None
c. 1, 2 and 4 only
d. 1, 2, 3 and 4 9) Great Barrier Reef sometimes seen in the
news recently is located in which of the
following?
6) Consider the following statements with
a. Tasman Sea
respect to CITES
b. Timor Sea
1. It is an International agreement
between governments for preventing c. Arafura Sea
species extinction due to International
d. Coral Sea
Wildlife trade.
2. All Species that are listed under IUCN
Red List are protected under the 10) issoh is a ritual of burning grasslands,
Convention on International Trade in practised in some parts of which of the
Endangered Species of Wild Fauna and following regions?
Flora (CITES).
a. Punjab
Which of the statement(s) given above
is/are incorrect? b. Arunachal Pradesh

a. 1 only c. Himachal Pradesh

b. 2 only d. Andaman and Nicobar Islands

c. Both 1 and 2
d. Neither 1 nor 2 Answers
1. b
7) Resolution 1267 sometimes seen in the news  India had recently revoked the Most Favoured
recently, is associated with which of the Nation (MFN) status bestowed on Pakistan in
following? 1996.
a. United Nation Security Council  While the term MFN suggests special
b. United Nation General Assembly preference for the country given MFN status, it
actually means it would be treated equally
c. International Court of Justice as all others.
d. None of the above  According to the World Trade Organisation
rules, countries cannot normally discriminate
between their trading partners.
8) Consider the following statements with
respect to International Court of Justice (ICJ)  If one country is granted a trade concession
such as, for example, lower import duties, then
1. It is an intergovernmental International all WTO members must be extended the same
tribunal. concessions.
2. It is located in Hague, Netherlands.  This principle is known as the Most Favoured
3. India is not a party to the ICJ statute. Nation treatment.

Which of the statement(s) given above is/are  Despite repeated promises, Pakistan has
correct? never granted MFN status to India.
a. 2 only 2. a
b. 1 and 3 only  To ensure optimum use of National
Waterways, the Inland Waterways

42
www.shankariasacademy.com | www.iasparliament.com
Authority of India (IWAI) launched a new  In fact, the appeal filed by the Supreme Court
portal LADIS – Least Available Depth against public disclosure under RTI has been
Information System recently. pending in the Supreme Court since 2010.
 LADIS will ensure that real-time data on least 4. b
available depths is disseminated for ship/barge
and cargo owners so that they can undertake  Indian Air Force is set to demonstrate its
transportation on NWs in a more planned way. full combat and fire capabilities during the 7th
edition of the Fire Power Demonstration
 An assured depth of waterway is required for (FPD) ‗Exercise Vayu Shakti-2019‘ in the
seamless movement of vessels. Pokharan firing range close to Indo-Pak border
in Rajasthan to be held on 16 February, 2019.
 If real time information is made available
regarding LADs in stretches of various NWs, it  Akash missile firing, gun firing from Advanced
will help transporters by guiding them on the Light Helicopter and MiG 29‘s air-to-ground
suitability of time of movement. role will be demonstrated in the exercise.
3. d  First conducted at Tilpat range in Delhi in July
1953, it is held once in three years.
 According to RTI Act, 2005, the Chief
Information Commissioner and Information 5. a
Commissioners shall be persons of
eminence in public life with wide  Deep sea corals, also known as cold-water
knowledge and experience in law, science and corals don‟t require Sunlight.
technology, social service, management,  Corals need saltwater to survive and require a
journalism, mass media or administration and certain balance in the ratio of salt to water.
governance.
 This is why corals don‘t live in areas where
 In its recent judgment the SC had stated that rivers drain fresh water into the ocean
“official bias” in favour of bureaucrats and (―estuaries‖).
government employees was evident from the
very beginning of the process for appointment  Highly polluted dirty water from a flood crisis
of Chief Information Commissioners and in northern Australia has spread to parts of
Information Commissioners. the Great Barrier Reef, placing it under severe
stress.
 In fact, the selection committee, which
shortlists candidates for appointment, is itself  Scientists warned that this could starve corals
composed of government employees. of light and provide fodder for the predatory
crown-of-thorns starfish.
 The apex court directed the government
to look beyond bureaucrats and
appoint professionals from “all walks
of life”.
 The Supreme Court also lauded the role of the
Right to Information (RTI) Act as an ―integral
part of any vibrant democracy.‖
 But the apex court itself has refused to
come within the ambit of the
information transparency law for the
past one decade.
 The court has firmly resisted back-to-back
decisions of the Central Information
Commission (CIC) and the Delhi High Court to
open up to the RTI regime as far as the issues
of Collegium and judicial appointments
are concerned. 6. b

43
www.shankariasacademy.com | www.iasparliament.com
 A recent study published in the journal Science  The grasslands were burnt every year by these
has discovered that 28% of Species listed islanders.
under IUCN Red list are not protected
by the Convention on International Trade in  This ritual, called issoh, helped regenerate the
Endangered Species of Wild Fauna and Flora grass they needed to thatch their houses before
(CITES), the primary international framework the monsoon.
for preventing species extinction due to  The 2004 tsunami washed away almost every
international wildlife trade. traditional house on Chowra island.
 CITES is an international agreement  Further rehabilitation efforts have forced the
between governments. islanders into nuclear housing units
7. a and devastated the grasslands.

 In the wake of terrorist attack in Pulwama, the  As the grasslands diminished, so did the
Chinese foreign ministry has made it clear that cultural practices like issoh diminished.
it would not support India‘s efforts to persuade
the 1267 Committee of the UN Security
Council to list Mazood Azhar as a global 18-02-2019
terrorist.
1) Consider the following statements with
8. a respect to Black Bucks
 International Court of Justice is the principal 1. Normally, female blackbucks do not
judicial organ of the United Nations possess horns.
(UN).
2. It has been listed as least concerned in
 It was established by the United Nations IUCN Red list.
Charter, which was signed in 1945 in San
3. Stray dogs and barbed wires are
Francisco (United States), and began work in
threatening its survival.
1946 in the Peace Palace, The Hague
(Netherlands). Which of the statement(s) given above is/are
correct?
 All Members of the United Nations
are ipso facto parties to the Statute of a. 1 only
the International Court of Justice.
b. 1 and 3 only
9. d
c. 2 and 3 only
 The Great Barrier Reef is located in the Coral d. 1, 2 and 3
Sea and stretches along the state of
Queensland on Australia‘s north-eastern
coastline.
2) Which of the following is/are not the causal
 Highly polluted dirty water from the recent factor(s) of glacial melting in Hindu Kush-
flood crisis in northern Australia has spread Himalayan (HKH) glaciers?
to parts of the Great Barrier Reef, placing 1. Air Pollution
it under severe stress.
2. Deforestation
 Scientists have warned that this could starve
corals of light and provide fodder for the 3. Global Warming
predatory crown-of-thorns starfish. 4. Land-use changes
10. d 5. Increase in Snow fall
 Chowra is an 8 sq.km. Island on Select the correct answer using the code given
the Andaman sea, part of the Nicobar below
archipelago.
a. 5 only
 And on it live 1,350 people, the Nicobarese
islanders of Chowra. b. 4 and 5 only

44
www.shankariasacademy.com | www.iasparliament.com
c. 1, 2, 4 and 5 only 6) Abohar Wildlife Sanctuary
(AWS) sometimes seen in the news recently is
d. 1, 2, 3, 4 and 5
located in which of the following states?
a. Punjab
3) Which one of the following best describes
b. Uttar Pradesh
the term “Glacial Lakes” sometimes seen in
news recently? c. Rajasthan
a. They are formed when a glacier erodes the d. Telangana
land, and then melts within the hole it has
created
7) Consider the following statements with
b. They are formed when a wide meander from
respect to Hindu Kush Himalayas (HKH)
the main stem of a glacial river is cut off,
creating a free-standing body of water 1. It is one of the greatest mountain
systems in the world covered across
c. They are formed when water from the melted
eight countries including China and
glaciers gets stored in a depressed land surface
Thailand.
d. They are formed due to a difference in
2. The glaciers of HKH feed 10 of the
temperature, resulting in creation of isolated
world's most important river systems,
ice blocks over water bodies
including the Yellow and Mekong.
Which of the statement(s) given above is/are
4) SPHEREx mission sometimes seen in the correct?
news recently is _______
a. 1 only
a. NASA‘s Space Telescope
b. 2 only
b. ISS‘s airborne Earth Observatory
c. Both 1 and 2
c. China‘s mission to study stellar flare
d. Neither 1 nor 2
d. None of the above

8) Consider the following statements with


5) Consider the following statements respect to Kelps
1. The United Nations General Assembly 1. Kelps are plants that look similar to
and UNESCO have decided to celebrate brown algae.
2019 as the “International Year of the
2. They thrive in cold, nutrient-rich
Periodic Table of Chemical Elements
waters.
(IYPT2019)”.
Which of the statement(s) given above is/are
2. It has been proclaimed so, to celebrate
correct?
the 150th anniversary of the discovery
of periodic system by Dmitri Ivanovich a. 1 only
Mendeleev.
b. 2 only
Which of the statement(s) given above is/are
correct? c. Both 1 and 2

a. 1 only d. Neither 1 nor 2

b. 2 only
c. Both 1 and 2 9) Consider the following statements with
respect to Pangolin
d. Neither 1 nor 2
1. It is the most trafficked reptiles in the
world.

45
www.shankariasacademy.com | www.iasparliament.com
2. Indian Pangolin – one among the eight 2. a
pangolin species found across the world
is listed as critically endangered under  The International Centre for Integrated
the IUCN Red list. Mountain Development (ICIMOD), an
intergovernmental organisation focussed on
Which of the statement(s) given above is/are environmental and social change in Hindu
correct? Kush Himalaya (HKH), released the findings
of a report.
a. 1 only
b. 2 only  It suggests Global warming as the primary
reason for glacial melt in the HKH region.
c. Both 1 and 2
 It also says the melting could accelerate
d. Neither 1 nor 2 because of increased air pollution.
 Air pollution from the Indo-Gangetic Plains
10) Through which of the following processes also deposits black carbon and dust on the
energy can be extracted from wastes? glaciers, hastening the thaw.

1. Incineration  In general, deforestation, land-use


changes, changes in
2. Gasification precipitation and decrease in
3. Pyrolysis snowfall could also impact the rate of glacial
retreat.
4. Anaerobic digestion
3. a
Select the correct answer using the code given
below  The most worrying outcome of glacial
shrinking and retreat is the increase in the
a. 1 and 3 only formation and size of glacial lakes.
b. 1, 2 and 3 only  Glacial lakes are formed when a glacier
c. 2, 3 and 4 only erodes the land, and then melts within
the hole it has created.
d. 1, 2, 3 and 4
 This in turn could lead to glacial lake
outburst floods (GLOF) when there is an
Answers avalanche or an earthquake.

1. d  Satellite data shows that the number of such


lakes in the Hindu Kush-Himalayan region
 Blackbuck hunting and poaching are grew to 4,260 in a decade from 3,350 in 1990.
prohibited in India as per Schedule I of the
Wildlife Protection Act of 1972. 4. a

 Blackbucks used to cover the entire  NASA will launch a new space telescope in
subcontinent of India except for the Western 2023 that could provide a glimpse of the first
coast, but as the human population kept moments in the history of the universe, and
growing, their numbers began plummeting. explore how common are the ingredients for
life in our galaxy‘s planetary systems.
 The most striking feature of blackbucks is the
long, spiralling horns of the adult male which  The Spectro-Photometer for the History of the
have ridges from the base to almost the Universe, Epoch of Reionization and Ices
tips. Females do not possess horns. Explorer (SPHEREx) mission is a planned
two-year mission funded at USD 242 million,
 Stray cattle and dogs, barbed „cobra‟ according to NASA.
wire and changed land use patterns spell
trouble for the majestic Punjab‘s State animal:  SPHEREx will survey the sky in optical as well
Black Buck, which has been listed as least as near-infrared light which, though not visible
concern under the IUCN Red list. to the human eye, serves as a powerful tool for
answering cosmic questions.

46
www.shankariasacademy.com | www.iasparliament.com
5. c  It was predicted that ocean warming and
acidification can change microbes on the kelp
 1869 is considered as the year of discovery of
surface, leading to disease and potentially
the Periodic System by Dmitri Mendeleev.
putting fisheries at risk.
 2019 will be the 150th anniversary of the 9. d
Periodic Table of Chemical Elements and has
therefore been proclaimed the "International  Though many think of them as reptiles,
Year of the Periodic Table of Chemical pangolins are actually mammals.
Elements (IYPT2019)" by the United
Nations General Assembly and  They are the most trafficked mammal in the
UNESCO. world.

6. a  There are eight pangolin species – four


each in Africa and Asia (two of them are in
 In 2019, so far, as many as eight blackbucks India).
have died in the Abohar Wildlife Sanctuary
(AWS), and a majority of them succumbed to  Indian Pangolin one among the eight pangolin
their injuries, caused by barbed wires while species is listed under the endangered category
trying to escape from stray dogs. of IUCN Red list.

7. b  Obsession for its supposedly medicinal scales


in China is believed to have made the ant-
 The Hindu Kush Himalaya (HKH) spread over eating Chinese Pangolin, one of the two species
3,500 kilometres from Afghanistan in the found in South Asia, extinct in India.
west to Myanmar in the east.
10. d
 The HKH is one of the greatest mountain
systems in the world, covering eight countries:  Waste to Energy (WTE), is a term that is used
Afghanistan, Bangladesh, Bhutan, China, to describe various technologies that
India, Myanmar, Nepal and Pakistan. convert non-recyclable waste into usable
forms of energy including heat, fuels and
 The glaciers feed 10 of the world's most electricity.
important river systems, including the Ganges,
Indus, Yellow, Mekong and the Irrawaddy.  WTE can occur through a number of processes
such as incineration, gasification,
8. b pyrolysis, anaerobic digestion, and
landfill gas recovery.
 Kelp Forests are underwater ecosystems
formed in shallow water by the dense
growth of several different species known as
kelps.
 Kelps are actually extremely large brown
algae, although they look like plants.
 They thrive in cold, nutrient-rich waters.
 Kelp attaches to the seafloor and eventually
grows to the water‘s surface and relies on
sunlight to generate food and energy.
 Kelps live further from the tropics than coral
reefs, mangrove forests, and warm-water
seagrass beds, so kelp forests do not overlap
with those systems.
 Climate change could lead to declines of
underwater kelp forests through impacts on
their micro biome.

47
www.shankariasacademy.com | www.iasparliament.com
19-02-2019 1. It is almost double the size of Kohinoor
Diamond.
1) Consider the following statements with
respect to Price Monitoring and Research 2. It is now in possession with the
Unit (PMRU) sometimes seen in the news Government of India.
1. It was proposed by the National 3. It was found in India in 1884 and was
Pharmaceutical Pricing Authority secretly transported to England to avoid
(NPPA). heavy duties then in place for raw
diamonds.
2. It aims to track violation of prices of
essential drugs and medical devices Which of the statement(s) given above is/are
under the Central Drugs Standard correct?
Control Organization (CDSCO). a. 2 only
3. It will functions under the direct b. 1 and 2 only
supervision of Drug Controller General
of India (DGCI). c. 2 and 3 only
Which of the statement(s) given above is/are d. 1, 2 and 3
correct?
a. 1 only 4) CyPAD sometimes seen in the news recently
b. 1 and 2 only is a/an
c. 2 and 3 only a. Initiative of IMD to process and detect
Cyclones with a lesser forecasting time
d. 1, 2 and 3
b. LPG gas cylinder awareness programme
among PM-ujjwala beneficiaries
2) Consider the following statements with c. Cyber-crime awareness and detection centre
respect to Tagore award for Cultural inaugurated in Delhi
Harmony
d. Cycle campaign organized by the Ministry of
1. It is awarded annually and carries an Environment to promote cycling among youths
amount of Rs.1 Crore.
2. This award is open to all persons
regardless of nationality, race, 5) Consider the following statements with
language, caste, creed or gender. respect to Investigation Tracking System for
Sexual Offences (ITSSO)
3. Ministry of culture will select the
awardees and recommend it to the 1. It is an online module available for both
President who will confer the award to citizens and law enforcement agencies.
the recipients. 2. It will allow States to undertake real-
Which of the statement(s) given above is/are time monitoring and management for
correct? completion of investigation in rape
cases in 2 months.
a. 3 only
3. It was launched by the Ministry of Home
b. 1 and 2 only affairs.
c. 2 and 3 only Which of the statement(s) given above
d. 1, 2 and 3 is/are incorrect?
a. 1 only
3) Consider the following statements with b. 1 and 2 only
respect to Jacob Diamond c. 2 and 3 only
d. 1, 2 and 3

48
www.shankariasacademy.com | www.iasparliament.com
9) Consider the following statements with
respect to Pradhan Mantri Jan Vikas
6) Consider the following statements with
Karyakram (PMJVK)
respect to Safe City Project
1. It aims at improving socio-economic
1. It aims to provide universal accessibility
conditions of the minorities and to
for differently-able persons in the areas
reduce imbalances in the identified
of environment, transport and
minority concentration areas.
Information & communication eco-
system. 2. It will be implemented as a Centrally
Sponsored Scheme during the
2. It is a centrally sponsored scheme with
remaining period of 14th Finance
Centre and state contributing the funds
Commission.
in 60:40 ratio.
Which of the statement(s) given above is/are
Which of the statement(s) given above is/are
correct?
correct?
a. 1 only
a. 1 only
b. 2 only
b. 2 only
c. Both 1 and 2
c. Both 1 and 2
d. Neither 1 nor 2
d. Neither 1 nor 2

10) Consider the following statements with


7) Magnitude of Substance Use in
respect to Great Indian Hornbill
India Report sometimes seen in the news
recently was released by National Drug 1. It is endemic to Andaman & Nicobar
Dependence Treatment Centre (NDDTC) of the Islands.
AIIMS in collaboration with?
2. Disintergration of their habitats for
a. NITI Aayog agriculture actually turned out to be
beneficial for their population.
b. Ministry of Home affairs
Which of the statement(s) given above
c. Ministry of Health and Family Welfare
is/are incorrect?
d. Ministry of Social Justice and Empowerment
a. 1 only
b. 2 only
8) Consider the following statements with
c. Both 1 and 2
respect to Safe City Implementation
Monitoring (SCIM) portal d. Neither 1 nor 2
1. It will facilitate online tracking of
deployment of assets and infrastructure
created under the Safe City projects. Answers
2. It has been developed by the Ministry of 1. a
Home affairs.  Kerala has become the first State to set up a
Which of the statement(s) given above is/are price monitoring and research unit (PMRU) to
correct? track violation of prices of essential drugs and
medical devices under the Drugs Price
a. 1 only Control Order (DPCO).
b. 2 only  After Kerala, it is now the turn of Punjab and
c. Both 1 and 2 Gujarat to have PMRU, set up by the
National Pharmaceutical Pricing Authority in
d. Neither 1 nor 2 collaboration with the Department of Health
and State Drug Controller.

49
www.shankariasacademy.com | www.iasparliament.com
 The National Pharmaceutical Pricing  The finished gem, with 58 facets, weighing 185.
Authority (NPPA) had proposed such a 75 carats was stunningly beautiful, in its cut,
system for the States and the Union Territories clarity and colour. (Kohinoor weighed only
five years ago. 105.6 carats).
 PMRU is aimed at monitoring the notified  It is a part of Nizam of Hyderabad‘s jewellery
prices of medicines, detection of violation of collection which was purchased in 1995 by
the provisions of DPCO, pricing compliance the Government of India at a cost of Rs.
and ensuring availability of medicines, among 218 Crore.
other objectives.
 Recently, it was kept in the exhibition of rare
 PMRU‘s will function under the direct collection of Nizam‟s jewels and jewellery at
supervision of State drug controller. National Museum, New Delhi.
2. b 4. c
 Tagore Award for Cultural Harmony was  To better equip the country in the fight against
instituted by the Government of India from cybercrime, Union Home Ministry has recently
2012 recognizing the contributions made by inaugurated a national cyber forensic lab and
Gurudev Rabindranath Tagore to humanity at the Delhi Police‟s cybercrime unit
large with his works and ideas, as part of the „Cyber Protection Awareness and
Commemoration of his 150thBirth Detection Centre (CyPAD)‟.
Anniversary in 2012, for promoting values
of Cultural Harmony.  The CyPAD is a modern, comprehensive
facility that aims to provide cyber
 It is awarded annually and carries an investigation, cyber forensics, cyber safety and
amount of Rs. One Crore (convertible to security-related services to the citizens as
foreign currency), a citation in a Scroll, a well as to police units and agencies of Delhi.
Plaque as well as an exquisite traditional
handicraft / handloom item.  It is the first cybercrime awareness and
detection centre opened in the country.
 A committee which selects the awardee
5. a
comprises of Prime Minister of India,
Chief Justice of India, Leader of  The Investigation Tracking System for Sexual
Opposition and two eminent Offences (ITSSO) is meant for Law
personalities. Enforcement Agencies in the country.
 The Award is open to all persons regardless  ITSSO is an online module available to law
of nationality, race, language, caste, creed or enforcement agencies at all levels- National,
gender. State, District and Police Station
 The President of India had recently presented  It allows State to undertake real-time
the Tagore Award for Cultural Harmony for monitoring and management for completion of
the years 2014, 2015 and 2016 to Shri investigation in rape cases in 2 months.
Rajkumar Singhajit Singh, Chhayanaut (a
cultural organization of Bangladesh) and Shri  It leverages the existing CCTNS data base,
Ram Vanji Sutar respectively. which covers nearly 15000 police stations in
the country.
3. b
Background
 Originally known as Victoria Diamond, Jacob
diamond was found in Kimberly mines in  The Criminal Law Amendment in April 2018
South Africa in 1884 and was secretly prescribes stringent penal provisions including
transported to England to avoid heavy duties death penalty for rape of a girl below the age of
then in place for raw diamonds. 12 years.

 It is almost double the size of Kohinoor  For swift administration of justice in such
Diamond (in the present shape). cases, the Act also inter-alia mandates
completion of investigation and trials within 2
months.

50
www.shankariasacademy.com | www.iasparliament.com
 To enhance women safety in the country,  SCIM also creates a digital repository of assets,
Government has formulated a multi-pronged infrastructure and social outreach programs,
action plan and initiated a series of measures. as well as best practices achieved in each City.
 ITSSO is one of such measures developed by 9. c
MHA towards Smart Policing.
 The erstwhile Multi-sectoral Development
6. b Programme (MsDP), a centrally sponsored
scheme has been restructured and renamed as
 In order to instil sense of security in Pradhan Mantri Jan Vikas Karyakram.
women in metro cities, Government has
identified eight cities for implementation of  The restructured Programme will be
Safe City projects in first phase at a cost of implemented during the remaining period
Rs.2,919 crore. of 14th Finance Commission i.e. up to
2019-20 as a Centrally Sponsored
 The cities are Ahmedabad, Bengaluru, Scheme.
Chennai, Delhi, Hyderabad, Kolkata, Lucknow
& Mumbai.  The PMJVK aims at improving socio-economic
conditions of the minorities and providing
 The project will be implemented under basic amenities to them for improving quality
the Nirbhaya Fund scheme as of life of the people and reducing imbalances
a Centrally Sponsored Scheme with in the identified minority concentration areas.
Centre and state contributing the funds in
the 60:40 ratio.  The projects to be taken up under PMJVK
would be related to creation of infrastructure
 It involves creation of on-ground assets, mainly in the sectors of education, health
resources & behaviour change programs for and skill development, besides innovative
safety of women. schemes for improving the socio-economic and
7. d living conditions of minority communities and
other communities living in the catchment
 National Drug Dependence Treatment Centre area.
(NDDTC) of the All India Institute of Medical
Sciences (AIIMS), New Delhi has submitted its 10. c
Report ―Magnitude of Substance Use in  The great hornbill (Buceros bicornis) also
India‖ sponsored by the Ministry of Social known as the great Indian hornbill, has a wide
Justice and Empowermentto Union distribution and can be found in China, India,
Minister for Social Justice and Empowerment Nepal, Bhutan, Bangladesh Myanmar,
recently. Thailand, Laos, Vietnam, Cambodia, Malaysia
8. c and Indonesia.

 In order to facilitate States to monitor and  The large majority of the population is found
manage the Safe City projects and avoid in India.
duplication on ground, an online Safe City
 A recent study of great Indian hornbills
Implementation Monitoring (SCIM)
nesting in the Anamalai hills which forms the
portal has been developed by Ministry of
southern portion of the Western Ghats has
Home Affairs.
discovered that these birds listed
 SCIM will facilitate online tracking of under vulnerable category of IUCN are
deployment of assets and infrastructure adapting to the changing environment.
created under the Safe City projects.
 Increased human activity in their belts,
 SCIM facilitates an evidence based online including agriculture, adds the pressure on
monitoring system. their habitatsand so their nesting sites need
special protection.
 Details on assets and infrastructure are
captured along with latitude and longitude  Disintergration of their habitats for agriculture
using GPS for greater transparency. actually turned out to be beneficial for Bengal
Florican population.

51
www.shankariasacademy.com | www.iasparliament.com
 Scientists found that the Bengal floricans in 1. It is a project under the Deendayal
Terai use not just the protected grasslands but Antyodaya Yojana - National Rural
the agricultural fields, too. Livelihoods Mission (DAY-NRLM).
 Floricans need alternating patches of short and 2. It will be implemented through loan
tall grass to thrive. assistance from the World Bank.

 Till several decades ago, the large herbivores of Which of the statement(s) given above is/are
the terai such as rhinoceroses and swamp deer correct?
would help in creating these perfect habitats by a. 1 only
clearing those grasslands.
b. 2 only
 But now there are fewer mega-herbivores left.
c. Both 1 and 2
 So this could be triggering this movement of
floricans into fields in search of short grass. d. Neither 1 nor 2

 This suggests that conserving these cultivated


areas could be as important as protecting the 4) Nyaya Bandhu App was recently launched
grasslands where these birds breed. by which of the following?
a. National Human Rights Commission
20-02-2019 b. Ministry of Panchayat Raj
1) Wallace Smith Broecker was renowned for c. Government of Assam
popularising which of the following terms?
d. None of the above
a. Crypto Currency
b. Global Warming
5) Consider the following statements with
c. Biodiversity respect to SWAYATT Initiative
d. Artificial Intelligence 1. It is an initiative to promote Start-ups,
Women and Youth advantage through
eTransactions on Government e
2) Consider the following statements with Marketplace (GeM).
respect to Guru Ravidas 2. It was launched by the Ministry of
1. He is the 8th Sikh guru and was the Commerce and Industry.
youngest of the 10 Gurus. Which of the statement(s) given above
2. Some of his poems were included in the is/are incorrect?
Adi Granth, the sacred scripture of a. 1 only
Sikhism.
b. 2 only
Which of the statement(s) given above is/are
correct? c. Both 1 and 2
a. 1 only d. Neither 1 nor 2
b. 2 only
c. Both 1 and 2 6) Recently, Ministry of Commerce and
Industry had issued a notification with respect
d. Neither 1 nor 2 to start-ups. Consider the following statements
in this regard
3) Consider the following statements with 1. An entity will be considered as a Start-
respect to National Rural Economic up up to a period of ten years from the
Transformation Project (NRETP) date of incorporation and registration.

52
www.shankariasacademy.com | www.iasparliament.com
2. An entity will considered as a Start-up, c. Supreme Court‘s advices to President on
if its turnover for any of the financial constitutional matters
years since incorporation and
d. Working of courts during vacation to
registration has not exceeded Rs. 100
adjudicate important cases
crore.
Which of the statement(s) given above is/are
correct? 10) SAFAR Mobile App sometimes seen in the
news recently was launched by which of the
a. 1 only
following?
b. 2 only
a. Ministry of Textiles
c. Both 1 and 2
b. Ministry of Civil aviation
d. Neither 1 nor 2
c. Ministry of Commerce and Industry
d. Ministry of Environment and Forest
7) Startup Runway sometimes seen in the
news recently, is an initiative of which of the
following? Answers
a. Department of economic affairs 1. b
b. Government e-market place  Wallace Smith Broecker raised early alarms
c. Department for Promotion of Industry and about climate change and popularised the term
Internal Trade ―global warming‖ has died recently.
d. NITI Aayog  He brought ―global warming‖ into common
use with a 1975 article that correctly predicted
rising carbon dioxide levels in the atmosphere
8) Consider the following statements with would lead to warming.
respect to National Tourism Advisory Council
 His term was a break with tradition. Earlier
(NTAC) studies of human impact on climate had called
1. It is a statutory body established it "inadvertent climate modification".
through National Tourism Development
 This was because while many scientists
Act 2003.
accepted that human activities could cause
2. It was initially established for a tenure climate change; they did not know what the
of 3 years and was later extended direction of change might be.
indefinitely.
2. b
Which of the statement(s) given above is/are
correct?  Guru Ravidas, a North Indian mystic poet of
the bhakti movement.
a. 1 only
 While the exact year of his birth is not known,
b. 2 only it is believe that the saint was born in 1377 C.E.
c. Both 1 and 2  Guru Ravidas Jayanti is celebrated on Magh
d. Neither 1 nor 2 Purnima, which is the full moon day in the
Hindu calendar month of Magha.
 Some 40 of the poems attributed to Ravidas
9) Pro bono legal services sometimes seen in were included in the Adi Granth (―First
the news recently, means? Volume‖), the sacred scripture of Sikhism.
a. Digitisation of all documents and legal services 3. c
in judiciary
 The Union Cabinet has recently approved the
b. Free legal aid for litigants who are unable to Implementation of an Externally Aided Project
afford it

53
www.shankariasacademy.com | www.iasparliament.com
namely ―National Rural Economic universe of Government Buyers by offering
Transformation Project (NRETP) under the products and services that are unique in
Deendayal Antyodaya Yojana – National Rural concept, design, process and functionality
Livelihoods Mission (DAY-NRLM) through through StartUp Runway corner.
loan assistance (IBRD Credit) from World
Bank.  DPIIT (Department for Promotion of Industry
and Internal Trade) certified Startups are
 The technical assistance provided by NRETP invited to offer their products/services for
and the higher level interventions facilitated by procurement on Government e Marketplace
the project will enhance the livelihoods (GeM).
promotion and access to finance and
8. d
scale-up initiatives on digital finance
and livelihood interventions.  The National Tourism Advisory Council
4. d (NTAC) serves as a think tank of the
Ministry of Tourism.
 Department of Justice, Ministry of law
and Justice has recently launched two new  The present NTAC was constituted on
mobile apps, Tele Law and Nyaya 20th October, 2016 under
Bandhu, that seek to offer easy access to the Chairmanship of the Minister of
justice and free legal advice across the country. Tourism with tenure of 3 years.

5. d  The committee constitutes important


Ministries, individual experts in the field of
 SWAYATT is an initiative to promote Start- travel and tourism management and Ex-offico
ups, Women and Youth Advantage Through members from the industry associations.
eTransactions on Government e Marketplace
(GeM). 9. b
10. c
 It will bring together the key stakeholders
within the Indian entrepreneurial ecosystem to  Union Minister for Commerce and Industry
Government e-Marketplace the national has launched the SAFAR Mobile App.
procurement portal.
 This app will help to gauge the actual issues
6. c faced by transporters on the road.
 A recent notification issued by the  The app tracks the incidence of a number of
government widened the definition of parameters faced by transporters and
start-ups to benefit a larger number of automatically records the location of the issue.
innovators and protect them from the tax.
 Transporters, truck drivers, and general public
 Now an entity will be considered as a Start-up may use the app for reporting their logistics
up to a period of ten years from the date of issues to the Ministry of Commerce and
incorporation and registration in place of Industry.
the earlier duration of 7 years.
 The app is available in 8 languages.
 Similarly, an entity will continue to be
recognised as a Start-ups, if its turnover for  SAFAR (System of Air Quality Weather
any of the financial years since incorporation Forecasting and Research) is also a
and registration has not exceeded Rs. 100 mobile application software.
crore in place of Rs. 25 crore earlier.  But, it was launched by the Ministry of
7. b Earth Science (MoES).
 StartUp Runway is a unique concept initiated
by Government e-Marketplace (GeM) in
association with Start -up India to promote
entrepreneurship through innovation.
 This program is an opportunity for agile
and lean Startups to reach out to the

54
www.shankariasacademy.com | www.iasparliament.com
21-02-2019 4) Consider the following statements with
respect to “Festival of India event” which was
1) Consider the following statements with held recently
respect to Operation Digital Board
1. It is a month-long festival celebrated by
1. Under the initiative, all classrooms of the Government of India to familiarise
Government Schools in India will be the younger generations with the
equipped with digital boards in the next cultural diversity of India.
five years.
2. It is organised by the Ministry of Culture
2. Department of School Education is the in association with Government of
sole implementing agency for this Odisha.
operation.
Which of the statement(s) given above is/are
Which of the statement(s) given above is/are correct?
correct?
a. 1 only
a. 1 only
b. 2 only
b. 2 only
c. Both 1 and 2
c. Both 1 and 2
d. Neither 1 nor 2
d. Neither 1 nor 2

5) The indelible ink that we used in our


2) Digidhan Mitra, a Chatbot launched elections to prevent double voting was
recently was designed and developed by which designed and developed by?
of the following agencies?
a. Bharat Electronics Limited
a. Department of Electronics
b. National Physical Laboratory
b. National Informatics Centre
c. Indian Institute of Chemical Biology
c. Election Commission of India
d. Mysore Paints and Varnish Limited
d. Central Board of Direct Taxes

6) Consider the following statements with


3) Consider the following statements with respect to Swine flu
respect to Technology Incubation and
Development of Entrepreneurs 2.0 (TIDE-2.0) 1. It requires swine as an intermediate
Scheme carrier to affect a person.
1. It provides financial as well as technical 2. Bedaquiline is the World Health
support to incubators engaged in Organisation (WHO) recommended
supporting ICT start-ups using medicine to treat Swine flu in India.
emerging technologies. Which of the statement(s) given above
2. It is implemented by the Ministry of is/are incorrect?
Commerce and Industry. a. 1 only
Which of the statement(s) given above is/are b. 2 only
correct?
c. Both 1 and 2
a. 1 only
d. Neither 1 nor 2
b. 2 only
c. Both 1 and 2
d. Neither 1 nor 2

55
www.shankariasacademy.com | www.iasparliament.com
7) Consider the following statements with c. Vernacular Press Act
respect to Sea Squirts
d. Jallianwala Bagh Massacre
1. It is a round; palm-sized animal
migrated based on the pollution of its Answers
habitat. 1. d
2. It could help measure plastic pollution  The Ministry of Human Resource
as it can filter tiny particles from the Development has recently launched
ocean and store them in its soft tissue. the Operation Digital Board (ODB) to
Which of the statement(s) given above is/are leverage technology in order to boost quality
correct? education in the country.
a. 1 only  It was launched on the lines of Operation
Blackboard to provide better digital
b. 2 only education in schools.
c. Both 1 and 2
 Under the initiative ODB, the digital board will
d. Neither 1 nor 2 be introduced all over the country
in government and government aided
schools from class 9th onwards as well as
8) United Nations General Assembly (UNGA) in higher education institutions in
proclaimed “2019” as the International Year of the next three years.
________
 University Grants Commission
a. Camelids (UGC) will be the implementing agency for
Operation Digital Board in Higher
b. Plant Health Education Institutions (HEIs).
c. Indigenous languages 2. b
d. Artisanal Fisheries and Aquaculture
 Designed and developed by National
Informatics Centre (NIC), the AI based
Digidhan Mitra enables a text & voice based
9) Hodeidah, Saleef, and Ras Issa ports conversation with the user, mining the
sometimes seen in the news were located in Digidhan Portal, to give customized
which of the following? information in graphical, tabular and textual
a. Red Sea format.
b. Persian Gulf  It provides bank wise transactions details as
well as growth pattern of various modes of
c. Indian Ocean
transactions like BHIM, IMPS, Cards etc. in
d. Mediterranean Sea tabular as well as graphical form.
3. a
10) Consider the following events  Department of Electronics and
Information Technology (DeitY) is
1. Rabindranath Tagore renounced his
implementing a scheme titled ―Technology
knighthood.
Incubation and Development of Entrepreneurs
2. Gandhi gave up the title of Kaiser-i- (TIDE)‖.
Hind, bestowed by the British for his
 It aims to assist institutions of higher learning
work during the Boer War.
to strengthen their Technology Incubation
The two events described above were to protest Centres and enable young entrepreneurs to
against which of the following events? initiate technology start-up companies for
commercial exploitation of technologies
a. Rowlatt Act
developed by them.
b. Partition of Bengal

56
www.shankariasacademy.com | www.iasparliament.com
 The Ministry of Electronics and Information close contact, including handshaking, hugging
Technology has launched the TIDE 2.0 and kissing.
Scheme recently at the VIVID 2019 event.
 Complications do not generally arise out of
 TIDE 2.0 has been devised to promote the swine flu. However, the elderly and ailing are
momentum into the tech entrepreneurship susceptible to complications.
landscape.
 In order to tackel the menace, the Drug
 The Scheme provides financial and Controller General of India has been asked to
technical support to incubators engaged in coordinate with drug manufacturers and
supporting ICT startups using emerging monitor the availability of OSELTAMIVIR,
technologies such as IoT, AI, Block-chain, the medicine recommended by WHO, in
Robotics etc. in seven pre-identified areas of various states.
societal relevance.
7. b
4. d
 Israeli researchers have recently found
 In Nepal, a month-long 'Festival of that Ascidians – round, palm-sized animals
India' has recently begun in Kathmandu. also known as Sea Squirts can thrive in dirty
industrial areas and pristine waters alike,
 It aims at familiarising the new generation allowing them to detect and analyze waste and
of Nepal about the similarities between the its impact in various regions..
Himalayan nation and India.
 Sea squirts are primarily sessile (permanently
 The event is organised by the Swami fixed to a surface), potato-shaped organisms
Vivekananda Cultural Centre, Embassy of found in all seas, from the intertidal zone to
India, Kathmandu and is supported by the the greatest depths.
Ministry of Culture, Government of India.
 It just sit in one place all their life and filter the
5. b water, like a pump.
 The indelible ink, a heady mix of chemicals,  These rubbery sea creatures with an irritating
dyes, aromatic materials and silver nitrate, is a habit of clinging to ships and invading
singular commodity that is exclusively used in beaches could help measure plastic
state-run elections from local bodies to the pollution as it can filter tiny particles from
Indian Parliament. the ocean and store them in its soft tissue.
 Mysore Paints and Varnish Limited  They can give a picture of what the whole reef,
(MPVL) is the sole supplier of indelible the whole ecosystem felt during its life.
ink for election purposes since 1962.
8. c
 MPVL is paying royalty annually to the state-
run National Research Development  In 2016, the United Nations General Assembly
Corporation (NRDC) for licensing the chemical adopted a resolution proclaiming 2019 as
formula developed by the Delhi- the International Year of Indigenous
based National Physical Languages.
Laboratory (NPL).
 International Mother Language Day is
6. c celebrated every year on the 21st of February
to promote linguistic and cultural diversity and
 The name Swine flu has persisted even multilingualism.
though the virus has long bypassed the
need for swine as an intermediate  Since 2019 is also the International Year of
carrier. Indigenous Languages, the theme of this year‘s
Mother Language Day is Indigenous
 H1N1 is a self-limiting viral, air-borne Languages as a factor in development,
disease spread from person to person, peace and reconciliation.
through large droplets generated due to
coughing and sneezing, indirect contact by  2024 – International Year of Camelids
touching a contaminated object or surface and

57
www.shankariasacademy.com | www.iasparliament.com
 2022 – International Year of Artisanal 1. It is one of the central armed police
Fisheries and Aquaculture forces and is the leading counter-
insurgency force in the Northwest India.
 2020 – International Year of Plant Health
2. They can arrest anyone and search a
 2019 – International Year of Indigenous place without warrant in the border
Languages districts of Assam, Arunachal Pradesh,
9. a Manipur, Nagaland and Mizoram.

 Yemen's government and Houthi rebels have Which of the statement(s) given above is/are
agreed on phase one of a mutual pullback of correct?
forces from the key city of Hodeidah. a. 1 only
 The deal on the first phase has been finalised b. 2 only
and they have agreed in principle for the
second phase. c. Both 1 and 2
d. Neither 1 nor 2
 Under phase one, forces from both sides will
withdraw from the ports of Hodeidah,
Saleef, and Ras Issa.
2) Consider the following statements
 A pullback from Hodeidah was part of
1. Shahpur-Kanti dam is built across the
December's ceasefire between Iranian-backed
River Sutlej.
Houthi rebels and the Saudi-supported Yemeni
forces. 2. River Ujh is a tributary of the River
Ravi.
Which of the statement(s) given above is/are
correct?
a. 1 only
b. 2 only
c. Both 1 and 2
d. Neither 1 nor 2

3) Consider the following statements with


respect to HOPE Portal which was launched
10. d recently

 The entire nation was stunned after the 1. It is an online platform established to
Jallianwala Bagh Massacre in 1919. digitalize and simplify the accreditation
process in Indian health care sector.
 Rabindranath Tagore renounced his
2. It aims to extend benefits associated
knighthood in protest.
with Insurance Regulatory and
 Gandhi gave up the title of Kaiser-i-Hind, Development Authority of India (IRDAI)
bestowed by the British for his work during the and Ayushman Bharat scheme to small
Boer War. scale hospitals across India.
Which of the statement(s) given above
is/are incorrect?
22-02-2019
a. 1 only
1) Consider the following statements with
respect to Assam Rifles b. 2 only
c. Both 1 and 2
d. Neither 1 nor 2

58
www.shankariasacademy.com | www.iasparliament.com
4) Consider the following statements with operate accreditation programme for
respect to Drone Olympics healthcare organisations in India.
1. It was conducted as a part of 2. It works under the Ministry of Health
International Drone Expo – 2019. and Family welfare.
2. It is being organized by the Ministry of Which of the statement(s) given above is/are
Defence. correct?
Which of the statement(s) given above is/are a. 1 only
correct?
b. 2 only
a. 1 only
c. Both 1 and 2
b. 2 only
d. Neither 1 nor 2
c. Both 1 and 2
d. Neither 1 nor 2
8) Consider the following statements with
respect to International Mobile Equipment
Identity (IMEI)
5) Consider the following statements with
respect to Quality Council of India (QCI) 1. It is a unique 15-digit number assigned
to every mobile phone.
1. QCI was set up jointly by the
Government of India and the Indian 2. Dual SIM mobile phones will have two
Industry. IMEI numbers, one for each SIM slot.
2. It is a non-profit autonomous 3. With the help of the IMEI, the mobile
organization under the Department for network can identify and prevent stolen
Promotion of Industry and Internal phones from being used.
Trade.
Which of the statement(s) given above is/are
Which of the statement(s) given above correct?
is/are incorrect?
a. 1 only
a. 1 only
b. 1 and 3 only
b. 2 only
c. 2 and 3 only
c. Both 1 and 2
d. 1, 2 and 3
d. Neither 1 nor 2

9) India‟s Trillion-Dollar Digital Economy


6) Padayani is a ritual theatre art form of Report is released by which of the following?
which of the following states?
a. Ministry of Finance
a. Gujarat
b. Ministry of Commerce and Industry
b. Maharashtra
c. International Telecommunication Union
c. Kerala
d. Ministry of Electronics and Information
d. Andhra Pradesh Technology

7) Consider the following statements with 10) Which of the following is/are located on
respect to National Accreditation Board for either side of the equator?
Hospitals and Healthcare Organizations
1. River Congo
(NABH)
2. Galapagos Islands
1. It is a constituent board of Quality
Council of India, set up to establish and 3. Ecuador

59
www.shankariasacademy.com | www.iasparliament.com
4. Venezuela  The Insurance Regulatory and Development
Authority of India (IRDAI) has mandated
Select the correct answer using the code given
hospitals to ensure a quality healthcare
below
ecosystem through National Accreditation
a. All except 4 Board for Hospitals and Healthcare
Organizations (NABH) Entry-Level
b. All except 2 and 4
Certification Process.
c. All except 3 and 4
 In order to extend benefits associated with
d. All of the above IRDAI and Ayushman Bharat scheme to small
scale hospitals across India, the government
has digitalized and simplified the NABH
Answers accreditation process.
1. b  The revised process is driven through a new
portal called HOPE - Healthcare
 The Assam Rifles, one of the central armed Organizations‟ Platform for entry-
police forces is the leading counter-insurgency level-certification.
force in the Northeast. It also guards the
sensitive India-Myanmar border.  It focuses to promote quality at nascent stages
by enrolling a wide range of hospitals across
 Assam Rifles personnel have been empowered the country including healthcare organizations
by the central government to arrest anyone (HCOs) and small healthcare organizations
and search a place without warrant in (SHCOs).
the border districts of Assam, Arunachal
Pradesh, Manipur, Nagaland and 4. b
Mizoram.  „Drone Olympics‟ held under the Aero-
 The Armed Forces (Special Powers) Act, which India 2019 event that held recently in
is operational in some parts of the Northeast, Bengaluru, was organized by the Ministry of
also gives similar powers to the Army Defence.
operating in the region.  It aims to encourage the UAV industry to
 Earlier, Assam Rifles was making arrests only connect with potential buyers and business
in areas where the Armed Forces (Special partners in the country and also provide an
Powers) Act was in effect. opportunity to the Armed Forces to assess the
capabilities of UAVs, the role of which is ever-
2. b increasing in the defence sectors, making them
 As per the Indus Waters Treaty, signed the ideal platform for intelligence,
between India and Pakistan in 1960, India has surveillance, reconnaissance, electronic
full rights to utilise the waters of the Ravi, warfare and strike missions.
the Beas and the Satluj rivers, while the 5. d
waters of the Indus, the Chenab and the
Jhelum belongs to Pakistan.  Quality Council of India (QCI) was set up
in 1997 jointly by the Government of
 A dam was started construction at Shahpur- India and the Indian
Kanti on the River Ravi. Industry represented by the three premier
 The project aims to minimise the wastage of industry associations i.e. Associated Chambers
water that now goes to Pakistan through the of Commerce and Industry of India
Madhopur Headworks downstream. (ASSOCHAM), Confederation of Indian
Industry (CII) and Federation of Indian
 The Ujh Project in Kathua, Jammu and Chambers of Commerce and Industry (FICCI),
Kashmir, aims to irrigate about 30,000 to establish and operate national accreditation
hectares and produce 300 MW electricity from structure and promote quality through
the waters of the Ujh, a tributary of the National Quality Campaign.
Ravi.
 The Department of Industrial Policy &
3. d Promotion, now known as DPIIT, Ministry of

60
www.shankariasacademy.com | www.iasparliament.com
Commerce & Industry, is the nodal ministry  The report finds that India can create up to $1
for QCI. trillion of economic value from the digital
economy in 2025, with about half of the
6. c
opportunity originating in new digital
 Padayani is a ritual theatre art form related ecosystems that can spring up in diverse
to the Goddess temples in the Central sectors of the economy.
Travancore regions of Kerala.
 Currently, India‘s digital economy generates
 Performed at night in the temple premises, the about $200 billion of economic value.
rituals of Padayani are a symbolic act for
10. a
pleasing the Goddess Bhadrakali whose fury
remains unabated even after her victory over  River Congo is the only river to cross the
the mythical demon, Darikan. equator twice.
7. a  The Galapagos Islands, part of the
Republic of Ecuador, are an archipelago of
 The National Accreditation Board for
volcanic islands distributed on either side of
Hospitals and Healthcare Organizations
the equator in the Pacific Ocean.
(NABH), a constituent body of Quality Council
of India (QCI), handling global
accreditation in Indian healthcare
sector under Ministry of Commerce and 23-02-2019
Industry. 1) Consider the following statements with
8. d respect to Indus Waters Treaty

 IMEI or ‗International Mobile Equipment 1. Under the treaty, all the waters of
Identity‘ is a unique 15-digit number assigned rivers, Ravi, Chenab and Beas were
to every mobile phone. allocated to India for exclusive use.

 IMEI is associated with a SIM slot, so dual SIM 2. The treaty has no provision for either
phones will have two IMEI numbers. country to unilaterally walk out of the
pact.
 Similarly, tablets with cellular connectivity will
also have IMEI number. Which of the statement(s) given above is/are
correct?
 With the help of the IMEI, the mobile network
a. 1 only
can identify and prevent stolen phones from
being used. b. 2 only
 When a mobile phone gets stolen, the user has c. Both 1 and 2
to submit his/her IMEI number to the network
d. Neither 1 nor 2
operator.
 They will immediately add it to a database
called the Central Equipment Identity 2) Consider the following statements with
Register (CEIR) and forward it to all respect to Seoul Peace Prize
network operators.
1. It was awarded biennially to reflect the
 This will block your mobile and prevent wishes of the people of Korea and to
anyone else from using it. crystallize their desire for ever-lasting
peace on earth.
 Even if the robber changes the SIM card,
he/she will still not be able to use it. 2. It was initially established to
commemorate the success of the 24th
9. d Olympic Games held in Seoul, Republic
 India‟s Trillion-Dollar Digital of Korea.
economy is a new report released
by Ministry for Electronics and
Information Technology(MEITY).

61
www.shankariasacademy.com | www.iasparliament.com
Which of the statement(s) given above is/are d. Odisha
correct?
a. 1 only
6) Consider the following statements
b. 2 only
1. No two recognised political parties can
c. Both 1 and 2 have the same symbol.
d. Neither 1 nor 2 2. A party that loses its recognition as a
state party or national party loses its
symbol immediately.
3) Consider the following statements with
Which of the statement(s) given above is/are
respect to Financial Action Task Force (FATF)
correct?
1. It is one of the 15 specialised agencies of
a. 1 only
United Nations (UN).
b. 2 only
2. Recently, FATF had listed Pakistan
under its „Black list‟ category. c. Both 1 and 2
Which of the statement(s) given above is/are d. Neither 1 nor 2
correct?
a. 1 only
7) Attukal Pongala is a famous festival
b. 2 only celebrated in which of the following states?
c. Both 1 and 2 a. Telangana
d. Neither 1 nor 2 b. Tamilnadu
c. Kerala
4) Consider the following statements with d. Maharashtra
respect to Repurpose Cooking Oil (RUCO)
Initiative
8) Consider the following statements with
1. It aims to collect and convert the used
respect to National Informatics Centre (NIC)
cooking oil into bio-fuel.
1. It is a premiere S&T institution of the
2. It was launched by the Food Safety and
Government of India providing
Standards Authority of India (FSSAI).
nationwide ICT infrastructure to
Which of the statement(s) given above is/are support e-Governance services and
correct? various initiatives of Digital India.
a. 1 only 2. It functions under the Ministry of
Electronics and Information
b. 2 only
Technology.
c. Both 1 and 2
Which of the statement(s) given above
d. Neither 1 nor 2 is/are incorrect?
a. 1 only
5) ImTeCHO Mobile App, which significantly b. 2 only
improves maternal health, is an initiative of
c. Both 1 and 2
which of the following states?
d. Neither 1 nor 2
a. Gujarat
b. Tamilnadu
9) According to the Wildlife (Protection) Act,
c. Kerala
1972, which of the following animals cannot be

62
www.shankariasacademy.com | www.iasparliament.com
hunted by any person except under some and Sutlej, while Pakistan has control over
provisions provided by law? the western rivers of Indus, Chenab and
Jhelum.
1. Leopard
 The wording of the treaty has no provision for
2. Nilgiri Tahr
either country to unilaterally walk out of the
3. Pygmy Hog pact.
4. Black Buck  Article XII of the IWT says, ―The provisions of
Select the correct answer using the code given this Treaty shall continue in force until
below terminated by a duly ratified
treatyconcluded for that purpose between
a. All except 1 the two governments.‖
b. All except 2 2. c
c. All except 1 and 4  Set up in 1990, the Seoul Peace Prize
d. All of the above is biennially awarded to personalities to
commemorate the success of the 24th
Summer Olympic Games held in Seoul.
10) Consider the following statements  The prize was established to reflect the wishes
1. In India, there are two species of of the Korean people and to emphasise on their
flamingos: Greater and Lesser desire for everlasting peace on earth.
Flamingo. 3. d
2. Greater Flamingos are widespread in  The Financial Action Task Force (FATF) is an
India and they migrate to South India inter-governmental body established in
during winter. 1989 by the Ministers of its Member
Which of the statement(s) given above is/are jurisdictions.
correct?  The objectives of the FATF are to set standards
a. 1 only and promote effective implementation of legal,
regulatory and operational measures for
b. 2 only combating money laundering, terrorist
c. Both 1 and 2 financing and other related threats to the
integrity of the international financial system.
d. Neither 1 nor 2
 Financial Action Task Force (FATF) has
recently decided to continue Pakistan in
Answers the „grey list‟, despite New Delhi‟s push to
blacklist the neighbour over terror funding.
1. b
4. c
 The Indus Water Treaty (IWT) is about sharing
 The Food Safety and Standards
of water of six rivers — Indus, Chenab, Jhelum,
Authority of India (FSSAI) has launched
Beas, Ravi and Sutlej — between India and
the Repurpose Cooking Oil (RUCO)
Pakistan.
initiative to collect and convert
 The water-sharing formula is governed by a used cooking oil into bio-fuel.
treaty the two countries signed in 1960 under
 Recently, Dehradun-based Indian Institute of
the leadership of the then Prime Minister of
Petroleum has successfully finished a pilot test
India, Jawaharlal Nehru, and Pakistan
to convert used cooking oil into bio-aviation
President, Ayub Khan.
turbine fuel (Bio-ATF), which can be blended
 The deal was brokered by the World Bank after with conventional ATF and used as aircraft
nine years of negotiation. fuel.
 Under the treaty, India has control over water 5. a
flowing in the eastern rivers — Beas, Ravi

63
www.shankariasacademy.com | www.iasparliament.com
 ImTeCHO or Innovative mobile-phone largest religious gathering of women on a
Technology for Community Health Operations single day when over 25 lakh women took part.
is an initiative of the Department of Health
8. d
and Family Welfare, Government of
Gujarat and is being implemented within  National Informatics Centre (NIC) was
existing primary health care system and established in 1976, and has since emerged as
centered on front line health workers. a ―prime builder‖ of e-Government / e-
Governance applications up to the grassroots
 ImTeCHO is an innovative mobile phone
level as well as a promoter of digital
application to improve performance of ASHAs
opportunities for sustainable development.
through better supervision, support and
motivation for increasing coverage of proven  It provides nationwide ICT infrastructure to
maternal, newborn and child health support e-Governance services and
interventions among resource-poor settings in various initiatives of Digital India.
India.
 It functions under the Ministry
6. d of Electronics and Information
Technology.
 Two or more recognised political parties can
have the same symbol provided they are 9. d
not contenders in the same State or
Union Territory.  All of these are protected under schedule-I of
Wildlife Protection Act, 1972.
 A party that loses its recognition
doesn‟t lose its symbol immediately. 10. c

 A party that is unrecognised in the present  In India there are two species of flamingos
election but was a recognised national or state 1. Greater Flamingo – Phoenicopterus rosues
party in not earlier than six years from the date
of notification of the election can be allotted its 2. Lesser Flamingo – Phoeniconaias minor
reserved symbol.  The Greater Flamingos are widespread in
 The extension in the use of symbol doesn‘t India, and they migrate to South India during
mean the extension of other facilities provided winter and spend their time in large reservoirs
to recognised parties like free time on and mud flats.
Doordarshal/AIR, free supply of copies of  First-ever count of flamingos is being
electoral rolls and so on. undertaken by Bombay Natural History
7. c Society (BNHS) in collaboration with NGOs
and local birdwatchers across the country.
 Attukal Pongala is one of the largest religious
congregations of women.  „Flamingo Count‟ will be conducted in two
phases at a pan-India level.
 Preparing ‗pongala‘ (a sweet offering) is
considered an auspicious all-women ritual as
part of the annual festival of the Attukal 25-02-2019
Bhagavathy Temple, which is popularly known
as the ―Women‘s Sabarimala‖. 1. According to National Tiger conservation
Authority, Nauradehi Wildlife
 As per the locals, the Pongala festival Sanctuary would be the suitable place for re-
commemorates the hospitality accorded by introduction of Cheetah, because
women in the locality to Kannagi, the
heroine of the Tamil epic Silappadhikaram 1. Its forests are very dense.
while she was on her way to Kodungallur in 2. The prey density for cheetahs is in
Kerala, after destroying Madurai city to avenge abundance
the injustice to her husband Kovalan.
Choose the correct alternative
 In 2009, the Pongala ritual had made it to the
Guinness Book of World Records for being the a. 1 only

64
www.shankariasacademy.com | www.iasparliament.com
b. 2 only Select the correct code
c. Both 1 & 2 a. 1 only
d. Neither 1 nor 2 b. 1 & 3 only
2. Consider the following statements regarding c. 2 & 3 only
Raja Ram Mohan Roy
d. All of the above
1. He was a staunch supporter of
monotheism.
5. Consider the following statements about
2. He is against the orthodox Hindu rituals
Ease of Living index 2018
and idol worship.
1. It is released by UN-Habitat .
3. His first book was named "Tuhfat al-
muwahhidin” . 2. It is to measure the liveability in both
rual and urban areas across nations.
Which of the above is/are correctly matched?
3. It tends to an evidence-based approach
a. 1 only
Which of the above statements is/are true?
b. 1 & 3 only
a. 2 only
c. 2 & 3 only
b. 3 only
d. All of the above
c. 1 & 3 only
d. All of the above
3. Consider the following statements regarding
Rutland Island.
1. It is one of the coral islands in 6. Which of the following is/are correctly
Lakshwadeep. matched
2. Jangil tribes are the indigenous peoples Isotopes Purpose
who occupy these Islands.
1. Cobalt-60 - Thyroid
Which of the above statement(s) is/are disorders
correct?
2. Iodine (I-131) - Detection of
a. 1 only cancer
b. 2 only 3. Phosphorus (P-32) - Reduction of
bone pain
c. Both 1 & 2
Select the correct code
d. All of the above
a. 1 & 2 only
b. 2 only
4. Which of the following
is/are incorrect about Financial action task c. 1 & 3 only
force (FATF) ?
d. 3 only
1. It has developed an international
standard for combating of money
laundering and the financing of 7. Consider the following statements about
terrorism and also the proliferation of Indian Council of Cultural relations
weapons of mass destruction.
1. The President of the council shall be
2. Organisation of Islamic Co-operation is appointed by the President of India and
also a member of FATF. shall hold office for three years.
3. FATF is associated with group of G-20 2. It is an autonomous organisation under
nations. Ministry of Culture.

65
www.shankariasacademy.com | www.iasparliament.com
Which of the above statements is/are true? b. 2 only
a. 1 only c. Both 1 & 2
b. 2 only d. Neither 1 nor 2
c. Both 1 & 2
d. Neither 1 nor 2 Answers
1. b
8. Consider the following statements regarding  Nauradehi was found to be the most suitable
forest fires in India area for the cheetahs as its forests are not very
1. Northeastern states have reported the dense to restrict the fast movement of
highest number of forest fire incidents Cheetahs.
in the country . 2. d
2. Pine trees have contributed to forest 3. b
fires in Western Himalayan states.
 It is located in Andaman
3. In central states, harvesting of mahua
flowers and extraction of tendu leaves 4. c
result in wildfires.
 Gulf Co-operation Council, not Organisation of
Which of the above statements is/are true? Islaminc Cooperation, is a member of FATF.
a. 1 & 2 only  It is assosciated with G7 Nations.
b. 2 & 3 only 5. b
c. 1 & 3 only  The Ministry of Housing and Urban Affairs
d. All of the above (MoHUA) launched a set of ‗Ease of Living'
standards that combines these various facets of
urban living.
9. Which of the following is correct about 6. d
Tokyo 2020 medal project?
7. a
a. A project to obtain more medals in Tokyo
Olympics by Indian youth  It is under the Ministry of External Affairs.

b. A project under Ministry of Sports & Youth 


Affairs 8. d
c. A project to use scrap metals in the  Northeastern states have reported the highest
manufacture of medals number of forest fire incidents in the country
d. A project to boost participation of Indian due to Shifting Cultivation.
citizens in manufacturing medals
 Pine trees have contributed to forest fires in
Western Himalayan states as the needles of
pine trees can easily catch fire.
10. Consider the following statements about
Mechi River.  In central states, harvesting of mahua flowers
for making local wine and extraction of tendu
1. It is a trans-boundary River between
leaves for making bidis result in wildfires.
India and Bangladesh.
9. c
2. It is a tributary of Mahananda River.
10. b
Which of the above statement(s) is/are
correct?  It runs between India and Nepal
a. 1 only

66
www.shankariasacademy.com | www.iasparliament.com
26-02-2019 c. 2 and 3 only
1) Consider the following statements with d. 1, 2 and 3
respect to Rail Drishti
1. It is a dashboard that brings 4) As per the National Survey on Extent and
information from various sources on a Pattern of Substance Use in India, consider the
single platform. following statements:
2. It will be available only to railway 1. Uttar Pradesh, Punjab, Haryana
employees. accounts for the highest number of
3. It was launched by the Ministry of people with opioid use problems.
Railways. 2. In terms of percentage of the population
Which of the statement(s) given above is/are affected, the highest proportions of
correct? opioid users are concentrated in parts
of the Northeast.
a. 3 only
Which of the statement(s) given above is/are
b. 1 and 2 only correct?
c. 1 and 3 only a. 1 only
d. 1, 2 and 3 b. 2 only
c. Both 1 and 2
2) Consider the following statements with d. Neither 1 nor 2
respect to Chagos Islands
1. It officially belongs to the United States
of America. 5) Hayabusa 2 sometimes seen in the news
recently is associated with which of the
2. It is located in the Pacific Ocean. following?
Which of the statement(s) given above is/are a. Eros
correct?
b. Bennu
a. 1 only
c. Ryugu
b. 2 only
d. Itokawa
c. Both 1 and 2
d. Neither 1 nor 2
6) Consider the following statements with
respect to National Action Plan – Viral
3) Consider the following statements with Hepatitis
respect to Nirbhay Cruise Missile 1. It aims to eliminate hepatitis by 2030.
1. It is India‟s first indigenously designed 2. It was launched by Ministry Health &
long-range sub-sonic cruise missile. Family Welfare.
2. It has a range of 295 kilometres and can Which of the statement(s) given above is/are
carry nuclear warheads of up to 300 kg. correct?
3. It is designed and developed by DRDO a. 1 only
in collaboration with Russia.
b. 2 only
Which of the statement(s) given above is/are
correct? c. Both 1 and 2
a. 1 only d. Neither 1 nor 2
b. 1 and 2 only

67
www.shankariasacademy.com | www.iasparliament.com
7) Consider the following statements with b. 2 only
respect to Global Digital Health Partnership
c. Both 1 and 2
Summit
d. Neither 1 nor 2
1. It is a global intergovernmental meeting
on digital health which aims to promote
safe use of technology by the children
and youths. 10) Diego Garcia is a U.S. military base located
in which of the following?
2. 4th Global Digital Health Partnership
Summit is hosted by the Ministry of a. Hawaai
Home affairs. b. Maluku Islands
Which of the statement(s) given above is/are c. Chagos archipelago
correct?
d. Galapagos Islands
a. 1 only
b. 2 only
Answers
c. Both 1 and 2
1. c
d. Neither 1 nor 2
 Ministry of Railways has recently
launched the Rail Drishti dashboard,
8) Which of the following factors is/are not the encompassing all the digitisation efforts in
cause(s) for the decline of insect population? Indian Railways and promoting transparency
and accountability.
1. Deforestation
 It brings information from various
2. Climate change sources on a single platform and gives
3. Use of pesticides access to key statistics and parameters
to every citizen of the country.
4. Intensive farming
2. d
Select the correct answer using the code given
below  The United Kingdom (U.K) must return
the Chagos Islands to Mauritius "as rapidly
a. 4 only as possible," the United Nations' highest court
b. 3 and 4 only International Court of Justice ruled recently,
branding its occupation of the Indian Ocean
c. 1, 2, 3 and 4 archipelago illegal.
d. None

9) Consider the following statements with


respect to Global Digital Health Partnership
(GDHP)
1. It is an international collaboration of
governments, government agencies and
multinational organisations.
2. It is dedicated to improve the health and
well-being of their citizens through the
best use of evidence-based digital
technologies. 3. a
Which of the statement(s) given above  Nirbhay is India‘s first indigenously designed
is/are incorrect? long-range sub-sonic cruise missile.
a. 1 only

68
www.shankariasacademy.com | www.iasparliament.com
 Equivalent to the famous American Tomahawk  The Ryugu asteroid is thought to contain
missile, Nirbhay missile is a two-stage relatively large amounts of organic matter and
missile which carries nuclear warheads of up water from some 4.6 billion years ago when
to 300 kg at a speed of 0.6 -0.7 Mach. the solar system was born.
 India had developed the Brahmos supersonic  It was launched in December 2014 and is
cruise missile with Russia but it had to go scheduled to return to the Earth with its
alone for developing the Nirbhay, which has samples in 2020.
a range of 1,000 kilometres.
 The probe will observe the surface of the
 This is because of the Missile Technology asteroid with its camera and sensing
Control Regime (MTCR), which forbids its equipment.
signatory countries from assisting or
6. c
providing technology to any other country
developing a cruise missile with a range of  National Action Plan – Viral
300 kilometres or more. Hepatitis was launched by Union
Ministry Health & Family Welfare.
 The range of Brahmos has thus been capped at
295 kilometres, just under the limit set by 7. d
MTCR.
 Global Digital Health Partnership Summit is a
4. c global intergovernmental meeting on digital
health.
 As per the National Survey on Extent and
Pattern of Substance Use in India, 2.06% of  GDHP aims to improve the health and well-
the population use opioids in any of 3 forms — being of citizens through the best use of
opium, heroin and pharmaceutical opioids evidence-based digital technologies.
(which includes a variety of medications).
 It is being hosted by the Ministry of Health
 The most commonly used is heroin is (1.14%), and Family Welfare in collaboration
followed by pharmaceutical opioids (0.96%) with World Health
and opium (0.52%). Organization (WHO) and the Global
Digital Health Partnership (GDHP).
 Of an estimated 77 lakh problem opioid users
(dependent or harmful use), more than half 8. d
are concentrated in just a few states.
 Habitat change is the biggest cause of insect
 Uttar Pradesh, Punjab, Haryana, population decline and extinction threat.
Delhi, Maharashtra, Rajasthan,
Andhra Pradesh and Gujarat account  Deforestation and changes in our land use,
for the highest number of people with including urban
opioid use problems. development and intensive farming,
cause significant losses and fragmentation of
 However, in terms of percentage of the pollinator-friendly habitats.
population affected, the highest
proportions are concentrated in parts  Pollution and the widespread use
of the Northeast (Mizoram, Nagaland, of pesticides in commercial agriculture are
Arunachal Pradesh, Sikkim, Manipur) along cited as the next big reasons.
with Punjab, Haryana and Delhi.  Pesticides can be harmful to insects that visit
5. c the plants.

 Hayabusa 2 is a Japanese probe sent to collect  Climate change also drives the extinction of
samples from an asteroid for clues about the insects in some regions.
origin of life and the solar system. 9. d
 It fired a bullet into the surface of  The Global Digital Health Partnership (GDHP)
the Ryugu asteroid. is an international collaboration of
governments, government agencies and
multinational organisations dedicated to

69
www.shankariasacademy.com | www.iasparliament.com
improving the health and well-being of their 2. It was ratified by U.N countries and
citizens through the best use of evidence-based came in to force since 2013.
digital technologies.
Which of the statement(s) given above is/are
 Governments are making significant correct?
investments to harness the power of
a. 1 only
technology and foster innovation and public-
private partnerships that support high quality, b. 2 only
sustainable health and care for all.
c. Both 1 and 2
 The GDHP facilitates global collaboration and d. Neither 1 nor 2
co-operation in the implementation of digital
health services.
10. c 3) Consider the following statements with
respect to Prompt Corrective Action (PCA)
 Britain must give Mauritius control of Framework
the Chagos Archipelago in the Indian
Ocean, the ICJ has said recently. 1. It is applicable only to Public Sector
Banks and not extended to Private
 The Chagos Islands are home to the US Sector Banks.
military base of Diego Garcia, under
lease from the United Kingdom since the 2. All banks kept under PCA are subjected
1960s. to both mandatory and discretionary
restrictions.
Which of the statement(s) given above is/are
27-02-2019 correct?
1) Consider the following statements with a. 1 only
respect to Bringing Green Revolution in
Eastern India (BGREI) programme b. 2 only

1. It aims to address the constraints c. Both 1 and 2


limiting the productivity of rice based d. Neither 1 nor 2
cropping systems in eastern India
including Assam.
2. It is now a sub-scheme of Rashtriya 4) Formjacking sometimes seen in the news
Krishi Vikas Yojana. recently is?

3. National Rice Research Institute (NRRI) a. A type of hacking technique that steals
is the nodal agency for planning, classified, sensitive government data
implementation and monitoring of this b. A type of hacking that targets retail websites
programme. and steal online shopper‘s details
Which of the statement(s) given above is/are c. A type of harassment where attacker harasses
correct? a victim using electronic communication
a. 3 only d. None of the above
b. 2 and 3 only
c. 1, 2 and 3 5) VISION-2040 sometimes seen in the news
d. None recently, is related with which of the following
sectors?
a. Aviation
2) Consider the following statements with
respect to Comprehensive Convention on b. Mines
International Terrorism (CCIT) c. Agriculture
1. It was proposed by India in 1996. d. Atomic Energy

70
www.shankariasacademy.com | www.iasparliament.com
c. 2 and 3 only
6) Consider the following statements with d. 1, 2 and 3
respect to Quick Reach Surface-to-Air missiles
(QRSAM)
9) Consider the following statements with
1. It was developed by DRDO, India, in
respect to Mirage 2000 jet
collaboration with Israel‟s IAI.
1. It is a multirole, single-engine fourth-
2. It has been developed by India to
generation fighter jet manufactured by
replace the „Akash‟ missile defence
United Kingdom.
system.
2. It was used by India since Kargil war,
Which of the statement(s) given above is/are
1999.
correct?
Which of the statement(s) given above is/are
a. 1 only
correct?
b. 2 only
a. 1 only
c. Both 1 and 2
b. 2 only
d. Neither 1 nor 2
c. Both 1 and 2
d. Neither 1 nor 2
7) Consider the following statements
1. It is also called as Silent Disaster.
10) WhatsFarzi is an app launched by IIIT-
2. They occur during the pre-monsoon Delhi students for which of the following
summer season. purposes?
3. They are more frequent over the Indo- a. To detect fake news
Gangetic plains of India.
b. To protect endangered species
Identify the disaster that correctly matches
c. To link farmers with end consumers
with the above description
d. To help students to overcome suicidal thoughts
a. Hailstorms
b. Heat waves
c. Blizzards Answers
d. Droughts 1. c
 The programme of ―Bringing Green Revolution
to Eastern India (BGREI)‖ was launched in
8) Consider the following statements with 2010-11 to address the constraints limiting the
respect to Sampriti Exercise productivity of ―rice based
1. It is a joint military exercise between cropping systems‖ in eastern India
India and Nepal. comprising seven (7) States namely; Assam,
Bihar, Chhattisgarh, Jharkhand, Odisha,
2. It will be held alternatively in both Eastern Uttar Pradesh (Purvanchal) and West
countries. Bengal.
3. Sampriti 2019 will be held in Nadia,  The programme is now a sub-scheme
West Bengal. of Rashtriya Krishi Vikas Yojana.
Which of the statement(s) given above  National Rice Research Institute
is/are incorrect? (NRRI) is the nodal agency for planning,
a. 2 only implementation and monitoring of BGREI
programme.
b. 1 and 3 only

71
www.shankariasacademy.com | www.iasparliament.com
2. a  The rationale for classifying the rule-based
action points into ―mandatory― and
―discretionary― is that some of the actions are
essential to restore the financial health of
banks while other actions will be taken at
the discretion of RBI depending upon
the profile of each bank.
4. b
 Formjacking is a new hacking technique that
targets online shoppers.
 Formjacking attacks are essentially a virtual
ATM skimming technique under which cyber
criminals target website of a retailer by
injecting malicious codes.
 These codes give away access to online
shoppers‘ payment details including sensitive
card details.
5. a
 Comprehensive Convention on International  Ministry of Civil Aviation has launched
Terrorism (CCIT) is a draft proposed by the document VISION2040 at the Global
India in 1996 that has yet to be adoptedby Aviation Summit 2019 that held recently.
the UN General Assembly.
6. b
 Recently, the Vice President of India has
stressed that, the UN must conclude the  India has successfully tested the indigenously
developed Quick Reaction Surface-to-Air
Comprehensive Convention on International
Terrorism (CCIT) with greater urgency and (QRSAM) short range missile recently.
should not delay the process over semantics.  It was indigenously developed by the
3. d Defence Research and Development
Organisation (DRDO).
 Under the revised PCA framework: Capital,
Asset quality and Profitability continue  QRSAM has a capability of engaging multiple
to be the key areas for monitoring. targets.

 Indicators to be tracked for Capital, asset  The missile has a range between 25-30 Kms,
quality and profitability would be and has been designed to be a quick reaction
CRAR/ Common Equity Tier I ratio, Net missile; it involves an all- weather weapon
NPA ratio (NNPA) and Return on Assets (RoA) system capable of tracking and firing.
respectively.  This missile has been developed by India to
 Leverage would be monitored additionally as replace the „Akash‟ missile defence
part of the PCA framework. system, and has 360-degree coverage, light
weight, high mobility and shorter second
 There are two types of reaction time as compared to ‗Akash‘.
restrictions, mandatory and
7. b
discretionary under PCA.
 Heat wave is a period of abnormally high
 Restrictions on dividend, branch expansion,
temperatures, more than the normal
director‘s compensation, are mandatory while
discretionary restrictions could include curbs maximum temperature that occurs during the
on lending and deposit. pre-monsoon (April to June) summer season.

 Based on each trigger point, the banks have to


follow a mandatory action plan.

72
www.shankariasacademy.com | www.iasparliament.com
 Heat -waves typically occur between March to 1. It was launched by National Co-
June, and in some rare cases even extend till operative Development Corporation
July. (NCDC).
 Heat waves are more frequent over the Indo- 2. The scheme has special incentives for
Gangetic plains of India. cooperatives of North Eastern region,
Aspirational Districts and also
 Heat wave is also called a ―Silent Disaster‖ cooperatives of women/SC/ST/PwD
as it develops slowly and kills and injures members.
humans and animals nationwide.
Which of the statement(s) given above is/are
8. b correct?
 Exercise Sampriti-2019 is an important a. 1 only
bilateral defence cooperation endeavour
between India and Bangladesh and this b. 2 only
will be the eighth edition of the exercise c. Both 1 and 2
which is hosted alternately by both
countries. d. Neither 1 nor 2

 Sampriti-2019 will be conducted at Tangail,


Bangladesh from 02 March to 15 March 2) Consider the following statements with
2019. respect to SHREYAS scheme
9. b 1. It aims to provide industry
 Recently, Indian Air Force Mirage 2000 apprenticeship opportunities, primarily
fighters were used to destroy the Jaish-e- for technical graduates to improve their
Mohammad camps at Balakot along the Line of employability of students.
Control. 2. It is a programme basket comprising the
initiatives of Ministry of Human
 It is a French multirole, single-engine
Resource, Ministry of Skill Development
fourth-generation fighter jet.
and NITI Aayog.
 It is manufactured by Dassault Aviation. Which of the statement(s) given above is/are
 It gained prominence after their remarkable correct?
success rate in the 1999 Kargil war. a. 1 only
 It has a maximum climbing speed of 60,000 b. 2 only
feet per minute and is designed for all weather
penetration at low altitude. c. Both 1 and 2
10. a d. Neither 1 nor 2

 The students of Indraprastha Institute of


Information Technology (IIIT), Delhi have 3) Consider the following statements with
developed an app called WhatsFarzi(roughly respect to SATAT scheme
be translated as whatsfake).
1. It is an initiative aimed at providing
 The app will track content on the internet and sustainable alternative towards
check it with authentic websites to curb spread affordable transportation.
of misinformation otherwise known as fake
news. 2. It was launched by the Ministry of Road
Transport and Highways.
Which of the statement(s) given above is/are
28-02-2019 correct?
1) Consider the following statements with a. 1 only
respect to Yuva Sahakar Scheme
b. 2 only

73
www.shankariasacademy.com | www.iasparliament.com
c. Both 1 and 2 7) Consider the following statements with
respect to Shanti Swarup Bhatnagar Prize
d. Neither 1 nor 2
1. It is an annual award presented only to
women.
4) Consider the following statements with
2. The Prize is given each year for
respect to Van Dhan Scheme
outstanding contributions to Science
1. It is an initiative for targeting livelihood and Technology.
generation for tribals by harnessing
Which of the statement(s) given above is/are
non-timber forest produce.
correct?
2. It was launched by the Ministry of Tribal
a. 1 only
Affairs and TRIFED.
b. 2 only
Which of the statement(s) given above is/are
correct? c. Both 1 and 2
a. 1 only d. Neither 1 nor 2
b. 2 only
c. Both 1 and 2 8) The Scheme of the Forest Rights Act, 2006,
is in tune with which of the following
d. Neither 1 nor 2
covenants?
1. UN Universal Declaration on Human
5) The term Vasudhaiva Kutumbakam which Rights
means “the whole world is one big family”, is
2. UN Convention concerning Indigenous
found in which of the following texts?
and Tribal populations
a. Mundaka Upanishad
3. UN Declaration on Rights of Indigenous
b. Maha Upanishad People
c. Mandukya Upanishad 4. UN Convention on Biological Diversity
d. Chandogya Upanishad 5. Rio Declaration on Environment and
Development
Select the correct answer using the code given
6) Consider the following statements with below
respect to Drug Price Regulation in India
a. All except 1
1. Essential Medicines listed under the
National List of Essential Medicines b. All except 1 and 4
(NLEM) would automatically come
c. All except 4 and 5
under price control.
d. All of the above
2. National Pharmaceutical Pricing
Authority (NPPA) can also cap the price
of non-scheduled drugs.
9) Consider the following pairs
Which of the statement(s) given above is/are
1. Bandipur National Park – Karnataka
correct?
2. Phen Wildlife Sanctuary – Madhya
a. 1 only
Pradesh
b. 2 only
Which of the pair(s) given above is/are
c. Both 1 and 2 correctly matched?
d. Neither 1 nor 2 a. 1 only
b. 2 only

74
www.shankariasacademy.com | www.iasparliament.com
c. Both 1 and 2 system so that clear pathways towards
employment opportunities are available to
d. Neither 1 nor 2
students during and after their graduation.
 SHREYAS is a programme basket comprising
10) Which of the following railway zones was the initiatives of three Central Ministries,
recently announced by the Centre as a new namely the Ministry of Human Resource
zone for Indian Railways? Development, Ministry of Skill
Development & Entrepreneurship and
a. East Coast Railway Zone
the Ministry of Labour&
b. Konkan Railway Zone Employment viz the National
Apprenticeship Promotion Scheme (NAPS),
c. South Coast Railway Zone the National Career Service (NCS) and
d. East Central Railway Zone introduction of BA/BSc/BCom (Professional)
courses in the higher educational institutions.
3. a
Answers
 SATAT is an initiative by Ministry
1. c of Petroleum and Natural Gas.
 National Cooperative Development  It is aimed at providing a Sustainable
Corporation (NCDC) has introduced Yuva Alternative Towards Affordable
Sahakar-Cooperative Enterprise Support and Transportation (SATAT)
Innovation Scheme.
 It will promote the use of Compressed Bio-Gas
 This is a youth friendly scheme which will (CBG) production plants and make available
attract youths to cooperative business CBG in the market for use in automotive fuels.
ventures.
 It will be launched in association with PSU oil
 The scheme will be linked to Rs.1000 crore marketing companies, which invites potential
Cooperative Start-up and Innovation Fund entrepreneurs to set up CBG plants.
(CSIF).
 It would benefit vehicle-users, farmers and
 The scheme envisages 2% less than the entrepreneurs.
applicable rate of interest on term loan for the
project cost up to Rs 3 crore including 2 years 4. c
moratorium on payment of principal.  Van Dhan Scheme is an initiative for
 The scheme has special incentive for targeting livelihood generation for tribals by
cooperatives of North Eastern region, harnessing non-timber forest produce.
Aspirational Districts and cooperatives with  Under the scheme, 10 Self Help Groups of 30
women/SC/ST/PwD members. Tribal gatherers are being trained and
 All types of cooperatives in operation for at provided with working capital to add value to
least 1 year are eligible. the products, which they collect from the
Jungle.
2. d
 It also aims at build upon the traditional
 SHREYAS – Scheme for Higher knowledge & skill sets of tribals by adding
Education Youth for Apprenticeship technology & IT for value addition.
and Skills.
 It also focussed on setting-up of tribal
 SHREYAS is a programme conceived for community owned Van Dhan Vikas Kendras in
students in degree courses, primarily non- predominantly forested tribal districts.
technical, with a view to introduce
employable skills into their learning, promote 5. b
apprenticeship as integral to education and  The Maha Upanishad is a Sanskrit text and
also amalgamate employment facilitating is one of the minor Upanishads of Hinduism.
efforts of the Government into the education

75
www.shankariasacademy.com | www.iasparliament.com
The text is classified as  Any citizen of India engaged in research in
a Vaishnava Upanishad. any field of science and technologies up to the
age of 45 are eligible.
 It is notable for its teaching of “Vasudhaiva
Kutumbakam”, or ―the world is one family‖. 8. d
6. d
 Para 19 powers which NPPA can exercise
for setting the price cap of scheduled and non-
scheduled drugs was transferred to
Standing Committee on Affordable Medicines
and Health Products (SCAMHP) housed in the
NITI Aayog.

9. c
 Phen Wildlife Sanctuary is a popular buffer
7. b zone of Kanha national park, Madhya
Pradesh.
 The award is named after the founder Director
of the Council of Scientific & Industrial  It lies in Southern region of Kanha tiger
Research (CSIR) India, the late Dr (Sir) Shanti reserve, close to Madhya Pradesh and
Swarup Bhatnagar and is known as the ‗Shanti Chhattisgarh state borders.
Swarup Bhatnagar (SSB) Prize for Science and  It was declared as a wildlife sanctuary
Technology‘. in year 1983 by Government of Madhya
 The Prize is given each year for outstanding Pradesh.
contributions to science and technology.

76
www.shankariasacademy.com | www.iasparliament.com
 The Fauna at this sanctuary mainly consists of  A separate railway zone was one of the key
the Tiger, Leopard, Wild boar, Cheetal, promises made to Andhra Pradesh in the
Sambar etc. bifurcation act.
10. c  The new zone named as South Coast
Railway (SCoR) Zone.
 The Centre has recently announced the
creation of a separate railway zone in Andhra  ―As per item 8 of Schedule 13 (Infrastructure)
Pradesh with headquarters of the Andhra Pradesh Reorganisation Act,
at Visakhapatnam. 2014, Indian Railways was required to
examine establishing a new railway zone in the
Successor State of Andhra Pradesh.

77
www.shankariasacademy.com | www.iasparliament.com

Vous aimerez peut-être aussi